Feb 2000 - 1999

Ace your homework & exams now with Quizwiz!

Vacuum-compression therapy may be applied following whirlpool cleansing of the wound at this manufacturer prescribed settings to emphasize the vacuum phase of the cycle in an attempt to increase capillary filling. a. -0.10 bar (-75 mm Hg) of negative pressure for 90 seconds followed by 0.05 bar (38.5 mm Hg) of positive pressure for 30 seconds b. -0.10 bar (-75 mm Hg) of negative pressure for 60 seconds followed by 0.05 bar (38.5 mm Hg) of positive pressure for 90 seconds c. -0.10 bar (-75 mm Hg) of negative pressure for 90 seconds followed by -0.05 bar (-38.5 mm Hg) of negative pressure for 30 seconds d. -0.10 bar (-75 mm Hg) of negative pressure for 90 seconds followed by 0.05 bar (38.5 mm Hg) of positive pressure for 30 seconds e. -0.10 bar (-75 mm Hg) of positive pressure for 90 seconds followed by -0.05 bar (-38.5 mm Hg) of negative pressure for 30 seconds.

a. -0.10 bar (-75 mm Hg) of negative pressure for 90 seconds followed by 0.05 bar (38.5 mm Hg) of positive pressure for 30 seconds

99. Pain threshold is attained at this sound pressure level: a. 125 dB b. 115 dB c. 100 dB c. 120 dB e. 130 dB

a. 125 dB

To be admitted to the PT-OT examination, the applicant at the time of filing of application must satisfy the following requirements, EXCEPT: a. A foreign applicant can be admitted to the PT-OT examination even if his country of citizenship does not allow Filipino therapists to practice in his country on the same as the subjects/citizens of that country b. Clinical experience must include neurological musculoskeletal, pulmonary, cardiovascular, pediatrics and psychosocial functions c. He/she is at least twenty-one years of age d. Must have completed at least nine months of clinical internship in physical and occupational therapy in an adequate PT-OT dept of a certified hospital or clinic under the supervision of a competent and duly registered therapist e. Applicant must have received a degree from a school of physical and occupational therapy duly recognized by the government

a. A foreign applicant can be admitted to the PT-OT examination even if his country of citizenship does not allow Filipino therapists to practice in his country on the same as the subjects/citizens of that country

Ulnar nerve paralysis will; manifest the following conditions, EXCEPT: a. Abduction and adduction of all digits are not affected b. If the MCP joints are in a flexed position, IP joints can be extended using the extensor digitorum. c. The hypothenar group does not function. d. The extensor digitorum is capable of extending the IP joints if the MCP joints are stabilized in a flexed position. e. The 4th and 5% digits cannot be extended due to the absence of intrinsic muscles.

a. Abduction and adduction of all digits are not affected

While there are eight nerves that exit the cervical spine, there are only seven cervical vertebrae. Where does the first thoracic nerve exit? a. Above the second thoracic vertebra b. Below the second thoracic vertebra c. Above the first thoracic vertebra d. Below the first thoracic vertebra e. Below the seventh cervical vertebra

a. Above the second thoracic vertebra

The mitochondria synthesizes this neurotransmitter at the end-plate of the motor neuron: a. Acetylcholine b. Troponin c. Adenosine d. Myosin e. Actin

a. Acetylcholine

When a muscle is the principal agent in producing a desired movement, it is said to be a/an: a. Antagonist b. Synergist c. Prime mover d. Postural muscle e. Fixators

c. Prime mover

The normal neck shaft angle is: a. 115 b. 130 c. 125 d. 120 e. 110

c. 125

The following statements are true of the patella, EXCEPT: a. Adapted through evaluation and with the development of bipedal gait to become an integral part of the knee extensor mechanism with all of the characteristics of a "true joint" b. Forces transmitted from the patella to the femoral sulcus increase as knee flexion increases c. The patella femoral compression forces are greater than body weight during walking and further increased to 2.5. times body weight during such activities as stair climbing d. Normal function depends upon adequate stabilization which is provided by both active and passive elements of the extensor mechanism e. Acting as a fulcrum, its primary function is to increase the extending moment of the quadriceps femoris muscle

a. Adapted through evaluation and with the development of bipedal gait to become an integral part of the knee extensor mechanism with all of the characteristics of a "true joint"

These molecules supply energy to myofilaments to allow movement of the actin filaments toward the center of the A bands: a. Adenosine Triphosphate b. Calcium Ions c. Adenosine Diphosphatase d. AOTA

a. Adenosine Triphosphate

The following applies to an open-complicated-complete-oblique fracture: a. All of these b. Fracture has injured surrounding nerves and blood vessels causing additional complications c. The bone breaks and completely snapped d. Either one or both broken ends protrude through or communicates with the wound site e. The bone breaks at an angle

a. All of these

Which of the following statements is true of the Radial Artery? a. All of these b. It runs down the lateral side of the front of the forearm supplying muscles along its course c. It is principally concerned with supplying blood to the thumb and index finger, and in the formation of the deep-palmar arch d. It reaches the wrist at the base of the thumb where its pulsation is readily felt

a. All of these

During the phase of mitosis, 46 pairs of chromatids are separated, forming 46 daughter chromosomes: a. Anaphase b. Prophase c. Metaphase d. Telophase

a. Anaphase

Following statements characterize Brown-Sequard syndrome, EXCEPT: a. Anterolateral system damage results in loss of pain and temperature sensation on the same side of the body below level of the injury b. There is tactile discrimination on the same side of the body below the level of the injury with dorsal column damage c. Simple touch sensation may be unimpaired due intact dorsal columns opposite the lesion. d. Lateral column damage results in paralysis of muscles on the same side of body below the injury. e. With dorsal column damage, there is loss of position and vibratory sense.

a. Anterolateral system damage results in loss of pain and temperature sensation on the same side of the body below level of the injury

A patient sustains a deep laceration over the right anterior thigh after stumbling into a modality cart. The therapist's immediate response should be a. Apply direct pressure over the wound b. Apply Betadine to cleanse and sterilize the wound c. Apply ice and elevate the leg d. Apply sterile dressings e. Document the incident in the patient's chart

a. Apply direct pressure over the wound

This surgical procedure is an advantageous consideration if only one side of the knee is worn a. Arthroscopy b. Norkalb osteotomy c. Knee arthroplasty d. Osteotomy e. Brian's arthroplasty

a. Arthroscopy

A movement disorder resulting from pathological changes involving the cortex and basal ganglia and characterized by slow writhing movements of a wormlike character involving the extremities, trunk and neck: a. Athetosis b. Chorea c. NOTA d. Paralysis Agitans

a. Athetosis

The following statements describe work hardening programs, EXCEPT a. Attempts to meet the functional and physical needs of clients b. Emphasize physical conditioning through strengthening endurance, and coordination exercises c. Supposed to simulate or duplicate actual job takes d. Work oriented e. A job specific type of rehabilitation program that attempts to address vocational and behavioral needs of clients f. None of the above

a. Attempts to meet the functional and physical needs of clients

These fibers are concerned with the involuntary control of smooth muscles and glandular activities: a. Autonomic Fibers b. None of these c. Sensory Fibers d. Motor Fibers

a. Autonomic Fibers

Blood cells which are activated by antigen only when this is associated with another host cell: a. B-lymphocytes b. T-lymphocytes c. Eosinophils d. Neutrophils e. Basophils

a. B-lymphocytes

When developing a treatment program, the caregiver a. Base the treatment goals on the outcome and function. b. Establishes the functional diagnosis and outcome goals c. Prepares a written home program for the client and family member d. Evaluates the patient's response to treatment determines progress towards functional outcomes. e. Gathers data and information for documentation

a. Base the treatment goals on the outcome and function.

The causes of short step length and of low speed in the hemiplegics are as follows, EXCEPT: a. Below normal ipsilateral hip extensor activity during ipsilateral stance b. Weak hip flexor at toe-off and early swing c. Weak push-ff prior to swing d. Extensive deceleration of the leg in the late swing e. Above normal contralateral hip extensor activity during contralateral stance

a. Below normal ipsilateral hip extensor activity during ipsilateral stance

Functionally, anatomic actions determine muscle participation in postures and movements, additionally, muscles also participate in posture and movements according to limb and body position in relation to gravity, applied resistance, and velocity in motion: a. Both statements are true b. First statement is true, second statement is false c. Both statements are false d. First statement is false, second statement is true

a. Both statements are true

This muscle has an extensive origin from the whole lower half of the shaft of the humerus and inserted by a tendon into the tuberosity of the ulna below the coronoid process, its sole concern is flexion of the elbow joint a. Brachialis b. Brachioradialis c. Pronator teres d. Biceps brachii e. Coracobrachialis

a. Brachialis

Compared to the actions potential in ventricular muscle, the action potential of the Sinuatrial Node shows a. No plateau b. A slow velocity of upstroke c. A high velocity downstroke d. Rapid influx of sodium ions e. the T wave a. C and E b. B and C d. D and E d. A and c e. A and B

a. C and E

The middle trunk is made up only of one nerve root, which is: a. C7 b. C6 c. T1 d. C8 e. C5

a. C7

Decreases vascular resistance by inhibiting calcium mediated contraction of vascular smooth muscle: a. Calcium channel blockers b. Beta adrenergic blockers c. Diuretics d. Calcium channel blockers e. Angiotensin-converting (ACE) enzyme inhibitors

a. Calcium channel blockers

Blood flow velocity is least in: a. Capillaries b. Arteries c. Pulmonary artery d. Veins

a. Capillaries

The gate control theory of Melzack and Wall has been used to explain the effectiveness of conventional sensory level TENS in reducing pain. The following statements discuss the theory, EXCEPT a. Central transmission of pain impulses via large-diameter, a myelinated fiber is via inhibition of second order neurons located in the dorsal horn. b. Intramuscular receptors send afferent impulses via group Ill and IV fibers and the spinothalamic tract to brain-stem cardiovascular centers. c. The transmission rate of action potentials from peripheral nociceptors to the central nervous system can be modulated by convergence of other afferent inputs at the level of the spinal cord d. Some group lll and IV afferent mediate the pressure response to sustained isometric muscle contraction e. Transcutaneous electrical stimulation of group | and Il afferent fibers is thought to modulate central transmission of pain impulses carried by group lll and IV fibers

a. Central transmission of pain impulses via large-diameter, a myelinated fiber is via inhibition of second order neurons located in the dorsal horn.

Proximal side to side tremor found on heel thin or finger-nose testing: a. Cerebellar tremor b. Parkinsonian tremor c. Myoclonus d. Senile tremor

a. Cerebellar tremor

The somatic sensory examination is the most difficult and least reliable part of the evaluation. The following rules and generalizations will be helpful, EXCEPT: a. Consistency of findings of several sensory examination gives some assurance of validity b. Peripheral nerve lesions are often associated with diminished or absent sweating, dry skin, tropic changes in the nails and loss of subcutaneous tissue c. The test ordinarily used for the sensory examination are rather crude and the findings are imprecise, the more so the less complete the lesion d. Subjective symptoms of numbness and paresthesia are unfortunately much the same qualitatively whether they result from thalamic, spinal, radicular or peripheral neural deficits e. Cutaneous sensitivity is greater on the face, hand, forearms genitalia and feet. Avoid testing calloused areas

a. Consistency of findings of several sensory examination gives some assurance of validity

Following statements characterize pernicious anemia, EXCEPT: a. Degeneration of dorsal and lateral columns of the spinal cord including gray matter b. There is diminished two-point discrimination and joint sensation c. Loss of position and vibration sense in the legs and positive Romberg sign. d. Difficulty in walking and tingling sensations in the feet e. In later stages, muscle stretch reflex may disappear.

a. Degeneration of dorsal and lateral columns of the spinal cord including gray matter

Decrease in resting membrane potential, making it more positive. a. Depolarization b. Membrane denervation c. Hyperpolarization d. Action potential degradation e. Variable conductance

a. Depolarization

The chemical substance that will activate trypsinogen into trypsin is a. Enterokinase b. Protein c. C1-ion d. Intestinal lipase e. Hydrochloric acid

a. Enterokinase

Coagulopathies interfere with the normal mechanism of the blood and affect the normal blood clotting factor that can result in the following, EXCEPT: a. Excesses do not contribute to tissue infarction b. Deficits can contribute to bleeding abnormalities and hemorrhage c. Excesses can contribute thromboemboli d. Excesses can contribute to obstruction of blood flow to vital organs e. Deficits and excesses are pathologic and interfere with oxygen transport

a. Excesses do not contribute to tissue infarction

The only muscle arising on the dorsum of the foot and has not counterpart in the hand: a. Extensor Digitorum Brevis b. Peroneus Brevis c. Extensor Hallucis Longus d. Extensor Hallucis Brevis

a. Extensor Digitorum Brevis

The Radial Nerve innervates the following muscle a. Extensor Indicis Propius b. The Medial Lumbricals c. Two Radial Lumbricals d. Flexor Digitorum Communis e. Abductor Pollicis Brevis

a. Extensor Indicis Propius

Bound by these tendons, they comprise the anatomical snuff box. a. Extensor Pollicis Longus, Abductor Pollicis Longus and Extensor Pollicis Brevis b. Extensor Digitorum Communis, Extensor Indicis Propius and Extensor Carpi Radialis c. Abductor Pollicis Longus, Abductor Pollicis Brevis and Extensor Indicis Propius d. Extensor Pollicis Longus, Extensor Indicis Propius and Extensor Pollicis Brevis e. Extensor Carpi Ulnaris, Extensor Digitorum Communis and Extensor Pollicis Brevis

a. Extensor Pollicis Longus, Abductor Pollicis Longus and Extensor Pollicis Brevis

When digital muscles have their proximal attachment in the forearm or humerus, they are classified as a. Extrinsic b. Thenar c. Intrinsic d. Midtrinsic e. Hypothenar

a. Extrinsic

In this class of lever, force is applied at one end of the lever arm, the weight to be moved is at the other end the fulcrum is at some point intermediate between the two: a. First b. Second c. None of these d. Third e. Fourth

a. First

Muscles which contract statically to support some pain of the body against the pull of contacting muscles are called: a. Fixators b. Synergists c. protagonist d. neutralizers e. antagonists

a. Fixators

These muscles mainly flex the interphalangeal joint a. Flexor digitorum profundus and flexor pollicis longus b. Flexor digitorum superficialis and flexor carpi ulnaris c. Lumbricals and flexor pollicis longus d. Lumbricals e. Flexor digitorum superficialis and lumbricals

a. Flexor digitorum profundus and flexor pollicis longus

If your patient can't support his full weight on his legs but does have a reasonably good muscular coordination and arm strength, which crutch walking gait would you choose? a. Four-point gait b. Swing-through three-point gait c. Three-point-and-one gait (partial-weight-bearing) d. Three-point-gait (non weight bearing) e. Two-point gait

a. Four-point gait

According to the original Borg scale for rating of Perceived on a rating of 15 is described as: a. Hard b. Very, Very hard c. Very, Very light d. Fairly light e. light

a. Hard

The following characteristics describe Type | muscle fibers, EXCEPT: a. Higher myosin ATPase activity than the other fiber type. b. Resistant to fatigue, with high levels of oxidative enzyme activity and low levels of glycolytic enzyme activity c. Contained in motor units characterized by a low firing frequency or discharge rate d. Associated with extensive capillary density and a high concentration of myoglobin e. Low threshold for activation and slow twitch contraction

a. Higher myosin ATPase activity than the other fiber type.

Functional map of the motor cortex resembling an image of the body turned upside down and reversed left to right: a. Homonculus b. Brodmann's area c. Premotor cortex d. None of these

a. Homonculus

This test helps determine whether the patient is malingering when he states that he cannot raise his leg in that as the patient tries to raise his leg, he puts pressure on the calcaneus of his opposite leg to gain leverage, the patient is not really trying if he does not bear down as he attempts to raise his leg. a. Hoover test b. Kernig test c. Milgram test d. Straight leg test e. Valsalva maneuver

a. Hoover test

The digestion of food involves the process of a. Hydrolysis b. Conjugation c. Condensation d. Mastication e. Decarboxylation

a. Hydrolysis

Which of the following statements describe the Thyroid Gland? I. It produces thyroxin, a very important iodine compound. II. It regulates the relative amounts of calcium in the blood and bones. III. Absence of the thyroid gland results in a mentally defective dwarf known as a cretin. IV. Its anemic blood supply is derived from the two superior thyroid arteries only. V. It derives its profuse blood supply from the two superior thyroid arteries and two inferior thyroid arteries. a. I, III and V b. II, IV and V c. II, III and IV d. I, II and III e. II, III and V

a. I, III and V

Which of the following statements are TRUE: I. All of the Flexor Digitorum Profundus is supplied by the median nerve. II. The median nerve supplies all of the Flexor Pollicis Longus. III. Adjoining half of the Flexor Digitorum Profundus is supplied by the median nerve IV. The ulnar nerve supplies the lateral half of the Profundus. V. Median half of Profundus is supplied by the ulnar nerve. a. II, III and V b. II, III and IV c. I, II and V d. I and III only e. III and IV only

a. II, III and V

An individual with rheumatoid arthritis that has a functional capacity adequate to perform only a few or none of the duties of occupation or self-care is classified as Class: a. III b. IV c. V d. II e. I

a. III

The relationship of vertebral levels to cord segments end to spinal roots are as follows: a. In infancy, the tip of the spinal cord is at about the L4-L5 level but it ascends to the L2- L3 level in maturity b. There are eight cervical cord segments and root pairs but only 7 vertebrae c. The C8 root exits below the C7 vertebra, and from there down numbered roots leave below the correspondingly numbered vertebrae d. In the cervical region, the cord segments approximate the corresponding numbered vertebrae, and the roots leave the cord at nearly a right angle e. The C1 root has no sensory component

a. In infancy, the tip of the spinal cord is at about the L4-L5 level but it ascends to the L2- L3 level in maturity

The following statements describe the anterior abdominal wall, EXCEPT: a. In pregnancy, the spatial relationship between the superior and inferior muscle attachments are altered as maternal superior thoracic diameter is decreased b. Consists primarily of four paired muscles, with fibers directed vertically, horizontally, and obliquely c. The muscles have skeletal attachments on the thoracic cage and pelvis and via broad aponeurosis to both the thoracolumbar fascia and the rectus sheath d. Alterations in the spatial relationship of muscle attachment and muscle angle of insertion also alters the muscle's ability to produce torque e. Rectus abdominis muscles may move laterally during pregnancy and may remain separated in the immediate post-birth period

a. In pregnancy, the spatial relationship between the superior and inferior muscle attachments are altered as maternal superior thoracic diameter is decreased

Denervation hypersensitivity a. Is due to an increased in the number of receptors b. Is due to a decrease in the sensitivity of receptors. c. Is due to an increase in the amount of neurotransmitters released d. Results from avulsion of the nerve e. Is due to an increased sensitivity to neurotransmitter.

a. Is due to an increased in the number of receptors

In one leg standing, the gluteus medius of the supporting leg is performing: a. Isometric contraction b. Positive work c. Eccentric contraction d. No work

a. Isometric contraction

The following statements are generally accepted in improving muscle strength and endurance, EXCEPT: a. Isometric contractions close to maximal forces for long periods of time is necessary to increase muscle endurance b. Concentric contractions against light loads may increase endurance c. To improve endurance a muscle must maintain small forces for long periods of time d. A muscle must generate close to maximal forces for short periods of time to build muscle strength e. Isometric or even eccentric forceful contractions may be necessary for building strength.

a. Isometric contractions close to maximal forces for long periods of time is necessary to increase muscle endurance

These procedures stretch the spinal cord and reproduce pain. The patient is in supine and asked to forcibly flex his head onto this chest with his hands behind his head. The result may be pain in cervical spine, and occasionally in the low back or down the legs indicating meningeal irritation, nerve root involvement or irritation of the dural covering of the nerve root. a. Kernig test b. Milgram test c. Gaeslan's sign d. Hoover test e. Naffziger test

a. Kernig test

Excursion and increase in tension of the lumbosacral roots may be accomplished by the following demonstrations, EXCEPT: a. Lateral rotation of the hip on straight leg raising b. Straight leg with medial rotation of the hip. c. Dorsiflexing the ankle at the end of straight leg raising d. Flexing the trunk during straight leg raising e. Straight leg raising

a. Lateral rotation of the hip on straight leg raising

In this type of gating, protein channel gates are opened by the binding of another molecule with the protein causing a conformational change in the protein molecule that opens or closes the gate: a. Ligand gating b. None of these c. Voltage gating d. Both of these

a. Ligand gating

The line of fusion between the External Oblique is the: a. Lines alba b. Aponeurosis c. Inguinal ligament (of Poupart) d. falx inguinalis e. transverses abdominis

a. Lines alba

This type of traction temporarily immobilize an injured area through hands pulling on the injured body part: a. Manual Traction b. Reduction Traction c. Mobilization Traction d. Skin Traction e. Skeletal Traction

a. Manual Traction

The following characterizes straight posterior knee instability, EXCEPT: a. May be caused by hyperextension, when some other force is directed against the extended knee b. Structures involved are the anterior cruciate ligament and the miniscotibial portions of the capsule c. May be caused by hyperextension, when a weight falls on the extended joint d. Rupture is caused by a force directed against the flexed knee that strikes the anterior aspect of the tibial head and drives it rearward e. Primary structures involved are the posterior cruciate ligament and the posterior part of the capsule

a. May be caused by hyperextension, when some other force is directed against the extended knee

Informed consent includes the following, EXCEPT: a. May be given before, during of after the initial treatment. b. The client or surrogate may sign a document to document the process indicating authorization to the proposed treatment c. Known or potential risks associated with the treatment are explained, and the client must be able to ask questions and receive responses about any aspects of treatment d. Consent from a legally qualified surrogate is necessary if the patient has not reached the legal age of consent, or mentally confused. e. Provide sufficient information about the proposed and alternative treatment/s appropriate to the client's condition, to allow him to make a knowledgeable decision.

a. May be given before, during of after the initial treatment.

The following statements describe cumulative trauma disorders, EXCEPT: a. May be related to a single episode incident b. The posture maintained by the worker-including the length of time sustained must be addressed in conjunction with the repetitious motion c. A relatively minor stress, one which has been imposed and successfully sustained numerous times in the past, can result in a significant breakdown of the system d. A combination or series of stresses applied over a period of time from which adequate recovery does not occur

a. May be related to a single episode incident

The left and right cerebral hemisphere is separated by the a. Medial longitudinal fissures b. Anterior median fissures c. Central sulcus d. Parieto-occipital sulcus e. Lateral fissure

a. Medial longitudinal fissures

A therapist presents an educational seminar on cerebral palsy, where he discusses its numerous etiologies. The following are congenital etiology of cerebral palsy, EXCEPT: a. Meningitis b. Rubella c. Syphilis d. toxoplasmosis

a. Meningitis

Muscle contraction strength gradation can be accomplished in the following manner, EXCEPT: a. Motor neurons with greatest innervations ratio are activated initially activating few muscle fibers. b. Increasing the frequency of stimulation of individual motor units c. Simultaneously recruiting the number of activated motor unit d. Initially activating few muscle fibers. e. As each active muscle fiber is developing maximum tension, the percentage of time is increased.

a. Motor neurons with greatest innervations ratio are activated initially activating few muscle fibers.

The largest bone in the proximal carpal row which represents the floor of the anatomic snuffbox: a. Navicular b. Lunate c. Trapezium d. Triquetrum e. Capitate

a. Navicular

Not a true peripheral nerve, an evaginated fiber tract of the diencephalon: a. Optic nerve b. Trochlear nerve c. Oculomotor nerve d. Olfactory nerve

a. Optic nerve

Surgical removal of a piece of bone to realign bone and shift weight-bearing stress away from a worn area: a. Osteotomy b. Arthrodesis c. Debridement and synovectomy d. None of these e. Menisectomy

a. Osteotomy

After successfully passing the Board Examination, the following initials may be added after the name of the successful physical/occupational therapy examinee a. PTRP/OTRP b. PTR/OTR c. PTRP/ORT d. RPT/ROT e. RPT/OTRP

a. PTRP/OTRP

Concerns position sense, postural bone and movement when stimulated by deep pressure, quick stretch of tissues and vibration. a. Pacinian corpuscles b. Miessner's Corpuscles c. Golgi mazzoni corpuscles d. Merkel's disks e. Ruffini corpuscles

a. Pacinian corpuscles

A therapist positions a patient in prone to measure knee flexion. Range of motion may be limited in this position due to a. Passive insufficiency of the knee extensors b. Passive insufficiency of sacrospinalis group c. Active insufficiency of the knee extensors d. Passive insufficiency of the knee flexors e. Active insufficiency of the knee flexors

a. Passive insufficiency of the knee extensors

This gait determinant reduces the vertical rise of the center of gravity as it passes over the same leg and compensated for by increased knee and ankle flexion so the swinging leg can pass under the lowered hemipelvis: a. Pelvic list b. Lateral displacement of the pelvis c. Knee, ankle and foot rotation d. Pelvic rotation e. Knee flexion in stance

a. Pelvic list

This gait determinant reduces the vertical drop of the center of gravity during doublestance phase and by forward movement of the hip joint, increases the length of the step: a. Pelvic rotation b. Pelvic list c. Lateral displacement of the pelvis d. Hip/knee flexion/extension mechanism e. Knee flexion in stance

a. Pelvic rotation

Rupture of this ligament allows excessive backward movement of the tibia on the femur: a. Posterior Cruciate Ligament b. Lateral Collateral Ligament c. Medial Collateral Ligament d. Anterior Cruciate Ligament

a. Posterior Cruciate Ligament

The following statements describe power, EXCEPT: a. Power will be positive when the moment and angular velocity are in the opposite direction b. A positive power usually indicates that the muscle is generating mechanical energy (concentric contraction) c. Reflects rate of work performed at a given joint d. A negative power usually indicates that the muscle is absorbing mechanical energy (eccentric contraction) e. Calculated by taking the product of the moment and the joint angular velocity

a. Power will be positive when the moment and angular velocity are in the opposite direction

The most common major orthopedic procedure performed in the elderly population is total hip arthroplasty. The following statements are true, EXCEPT: a. Protecting the prosthesis from unnecessary forces does not retard premature loosening. b.Carrying a load in a single hand is an activity that likely places excessive force on the hip. c. Mechanical factors such as micro motion and characteristics of implant materials contribute to prosthetic loosening. d. Studies showed that loosening of cemented components occurred in 30 % to 40% of patients 10 years after surgery. e. Biologic factors, including sepsis, and osteolytic responses to debris from implant are implicated to contribute to prosthetic loosening.

a. Protecting the prosthesis from unnecessary forces does not retard premature loosening.

Following statements are true of upper motor neurons, EXCEPT: a. Provides final direct link with muscles through myoneural junctions b. Synapse directly on alpha, beta and gamma motor neuron in the spinal cord and cranial nerve nuclei. c. Completely contained within the central nervous system d. Synonymous to corticospinal tract e. Lesion results in exaggerated deep reflexes and muscle rigidity.

a. Provides final direct link with muscles through myoneural junctions

A therapist evaluates a patient with bicipital tendinitis. These clinical findings are expected to be identified, EXCEPT: a. Referred pain in the C7-C8 dermatome b. Isometric resistance to the biceps brachial increases subjective pain level c. A painful arc is noted with active range of motion to the involved shoulder d. Tenderness to palpation exists over the bicipital tendon

a. Referred pain in the C7-C8 dermatome

A therapist is treating a patient three days status post total hip replacement. The patient is in bed with a sling-pulley system. Assuming an uncomplicated postoperative course, the following exercises are appropriate, EXCEPT: a. Resisted abduction and adduction with the thigh supported in the sling b. Active knee flexion and extension in the suspension sling c. Bilateral ankle pumps d. Active exercise to the upper extremities

a. Resisted abduction and adduction with the thigh supported in the sling

Development of the vertebrate nervous system involves progressive neural events which can include the following, EXCEPT: a. Retraction of neural projections b. Formation of complex neural pathways c. Synaptogenesis d. Myelinization e. Axonal growth

a. Retraction of neural projections

Rh factor incompatibility occurs in this setting: a. Rh (-) mother, Rh (+) fetus, Rh (+) father b. Rh (+) mother, Rh (-) fetus, Rh (+) father c. Rh (-) mother, Rh (+) fetus, Rh (-) father d. Rh (+) mother, Rh (+) fetus, Rh (+) father e. Rh (-) mother, Rh (-) fetus, Rh (+) father

a. Rh (-) mother, Rh (+) fetus, Rh (+) father

The following entries in the assessment section of SOAP notes are all correct, EXCEPT: a. Right lower extremity muscle strength is within normal limits b. Patient should be independent with straight cane by discharge date c. Strength duration curves show radial nerve regeneration d. Patient continues to make steady progress with upper extremity resistive exercises

a. Right lower extremity muscle strength is within normal limits

Normal spread of excitation in the heart is: a. SAN —> AVN —> Bundle of his —> Purkinje Fibers —> Ventricles b. AVN —> Bundle of His —> Atrium —> Purkinje Fibers —> Ventricles c. SAN —> AVN —> Purkinje Fibers —> Bundle of His —> Ventricles d. SAN —> Bundle of His —> AVN —> Purkinje Fibers —> Ventricles e. AVN —> SAN —> Purkinje Fibers —> Ventricles —> Atria

a. SAN —> AVN —> Bundle of his —> Purkinje Fibers —> Ventricles

Due to their inverted tripod arrangement, they function as a stabilizing mechanism for the pelvis and capable of co-operating with the "splint" mechanism of the iliotibial tract on the lateral side. a. Sartorius, Gracilis and Semitendinosus b. Adductor Magnus, Vastus lateralis, and Biceps c. Semitendinosus, Semimembranosus and gracilis . d. Semitendinosus, Biceps and Gracilis . Piriformis, e. Quadratus Femoris and Obturator Internus

a. Sartorius, Gracilis and Semitendinosus

The hamstrings are composed of: a. Semitendinosus and Semimembranosus b. Gluteus Maximus and Biceps Femoris c. all of these d. None of these

a. Semitendinosus and Semimembranosus

The following are actions of the upper fibers of the Trapezius: a. Shrugs the shoulders, maintain shoulder level but in fatigue allow the shoulders to droop b. Steady the scapula at the very beginning of the movement of raising the arm above the head c. Pull the medial end of the scapula downwards d. Cooperative with the horizontal fibers to rotate the glenoid, cavity upwards e. Retract the scapula

a. Shrugs the shoulders, maintain shoulder level but in fatigue allow the shoulders to droop

Of the total volume of water presented to gastrointestinal tract per 24 hours, the largest bulk is absorbed by the a. Small intestine b. Stomach c. Mouth d. large intestine e. esophagus

a. Small intestine

During phase O of the cardiac action potential, there is rapid influx of these ions: a. Sodium b. Oxygen c. Hydrogen d. Potassium e. calcium

a. Sodium

To test this nerve, the subject is asked to turn the head to the left and then to the right while testing strength of contraction of sternomastoid and trapezius muscle contraction by resisting movement: a. Spinal accessory nerve b. Vagus nerve c. Hypoglossal nerve d. Glossopharyngeal nerve

a. Spinal accessory nerve

Sends impulses to inhibitory interneurons to relax antagonist muscles. a. Spindle b. Annulospiral endings c. Motor impulses d. Golgi tendon organs e. Flower spray endings

a. Spindle

Assume a simulated muscle using two different moment arms, wherein muscle A's moment arm is much less than in muscle B. This means that a. Muscle A will change length much less for a given change in joint angle compared with the same change in joint angle in muscle B b. The active ROM for the muscle-joint in A will be much greater than, B, in spite of the fact that their muscular properties are identical a. Statements A and B are both false b. Statements A and B are both true c. Statement A is true but statement B is false d. Statement A is false but statement B is true

a. Statements A and B are both false

This joint is the upper extremity's only attachment to the trunk. a. Sternoclavicular b. Acromioclavicular c. Scapulothoracic d. Glenohumemral e. Scapulothoracic

a. Sternoclavicular

Mean arterial pressure is expected to increase if a. Stroke volume increases b. There is maintained exertion c. Calcium channel blockers are administered d. cardiac output decreases e. there is generalized vasodilatation

a. Stroke volume increases

The following statements apply to Duchenne (pseudohypertrophic) dystrophy, EXCEPT: a. Subject is still able to rise from a forward flexed position without using his hands to assist the movement b. Contractures are common and patients seldom survive into the third decade c. The enlargement of the muscles is due to fatty infiltration and the muscles are weak. d. A primary, genetic disorder characterized by early weakness and enlargement of the musculature of the calves, thighs, hips and shoulders e. Onset is in the first five years of life and occurs almost exclusively in boys

a. Subject is still able to rise from a forward flexed position without using his hands to assist the movement

Following are most frequently reported musculoskeletal or fibrous connective tissue symptoms, EXCEPT: a. Swelling and numbness of the extremities b. Swelling in articular, periarticular and soft tissue c. Nonrestorative sleep with morning fatigue d. Aches and pains that are specific, fluctuating through the full range of pain sensations frequently accompanied by marked stiffness e. Tender points frequently accompanied by muscle spasms or nodules that cluster in regions around the neck and shoulders, upper chest wall and lower back

a. Swelling and numbness of the extremities

In its most basic form, the sarcomere length-tension relationship illustrates that a. Tension generated in skeletal muscles is a direct function of the magnitude of overlap between the actin and myosin filaments. b. Muscle relative titanic tension is plotted as a function of sarcomere length. c. As muscle length decreased, overlap between actin and myosin was not possible, and the amount of tension generated by the muscle increased as sarcomere length decreased. d. At very long and very short lengths, muscles generate tension, whereas at optimal lengths, muscles generate higher tensions. e. Isometric contractions are performed at different lengths, and peak isometric tension is measured at each length.

a. Tension generated in skeletal muscles is a direct function of the magnitude of overlap between the actin and myosin filaments.

Muscle contraction occurs if a. Tension is generated in the muscle fibers resulting in shortening, lengthening or maintaining length. b. The muscle resists stretching. c. The muscle fiber shortens. d. The muscle is in concentric status. e. The angle of the joint either increases or decreases.

a. Tension is generated in the muscle fibers resulting in shortening, lengthening or maintaining length.

In retinacular test, the proximal IP joint is held in a neutral position and the distal IP joint attempts to move into flexion. When the proximal IP joint is flexed to slightly relax the retinaculum, and the distal IP joint still does not flex a. The distal IP joint capsule is probably contracted b. There is no contraction in the distal IP joint capsule c. The retinacular ligaments are loose d. The retinacular ligaments are tight e. None of these

a. The distal IP joint capsule is probably contracted

A synovial joint is characterized by the following distinguishing feature/s, EXCEPT: a. The hyaline cartilage is rich in blood vessels b. Articular cartilage reduces to minimum friction between the articular capsules. c. A lubricated articular cartilage d. A potential cavity e. A capsule of fibrous tissue line with synovial membrane.

a. The hyaline cartilage is rich in blood vessels

In Thomas test, shortened two-joint muscles are diagnosed when a. The knee flexion ability is good but the thigh rises up off the table. b. The leg is able to maintain contact with the table but the knee cannot flex past 70 degrees. c. The thigh rises off the table and the knee is unable to flex past 70 degrees d. The posterior thigh of the first leg can lay flat on the table with approximately 80 degrees of knee flexion. e. None of these

a. The knee flexion ability is good but the thigh rises up off the table.

The following statements characterize the hip abductors (HA), EXCEPT: a. The main function of the muscles is to provide sagittal place stability for the hip during the single-limb support phase of walking. b. The HA muscles must produce a torque large enough to match the torque produced by the body weight to achieve frontal plane stability. c. Reducing the need for excessive forces from the HA muscles should minimize the forces produced across he hip. d. The sum of the HA muscle derived force plus the force of body weight may reach 3 to 3.5 times body weight during mid stance. e. The force produced by the HA muscles is the largest contributor to the prosthetic hip reaction force.

a. The main function of the muscles is to provide sagittal place stability for the hip during the single-limb support phase of walking.

The following statements are true of the shoulder joint, EXCEPT: a. The movement primarily involves the clavicle and scapula b. The movement occurs between the scapula and the humerus c. The joint is a poor "fit" and a lax capsule d. It is multi-axial ball-and-socket joint with a remarkable degree of freedom of movement e. Its extreme mobility is achieved at the expense of stability and security

a. The movement primarily involves the clavicle and scapula

The following are true of the structural foundation of the knee, EXCEPT: a. The osseous portions of the knee are the femur, tibia, patella, and fibula b. The added depth of the tibial plateau is extremely important because the lateral femoral condyle and lateral tibial plateau are both somewhat convex c. The proximal end of the tibia flares to create a plateau with anterior and posterior section divided by the tibial spines d. The menisci deepen the contour of the tibial plateau to provide a good seat for the corresponding femur condyles e. The shape of the femoral medial and lateral condyle is important in the movement of the tibia on the femur

a. The osseous portions of the knee are the femur, tibia, patella, and fibula

The following statements describe the anterior compartment on the leg's anterolateral side between the tibia and the fibula, EXCEPT: a. The tibialis anterior, extensor hallucis longus and the extensor digitorum longus inserts, into this compartment b. The anterior fascia, posterior tibia, fibula, and interosseous ligament render it inflexible and unyielding c. If the structures are tight and intractable, and if the palpation elicits tenderness, there is evidence of an anterior compartment syndrome. d. Because of its inability to expand, swelling in the anterior compartment can create foot drop or anterior compartment syndrome e. Necrosis of the muscles, nerves and vessels may result from pathology that may cause swelling within the anterior compartment

a. The tibialis anterior, extensor hallucis longus and the extensor digitorum longus inserts, into this compartment

The following statements characterize idiopathic Parkinson's Disease, EXCEPT: a. There is progressive accumulation of dopaminergic neurons in the brainstem nuclei b. As disease advances, motor fluctuations frequently emerge c. A chronic neurological condition d. There is reduction in dopamine receptor sites in the striatum e. Swings in motor performance associated with changing levels of dopamine uptake can result un akinesia, hypokinesia, rigidity, dyskinesia and dystonia

a. There is progressive accumulation of dopaminergic neurons in the brainstem nuclei

Assistive devices are often used and prescribed to facilitate the process of gait training. The following statements are true, EXCEPT: a. These devices may be used as permanent functional aids the can be used for home or community ambulation. b. When a person using an assistive device has compromised cardio respiratory function, the increased demand may not be within the limits of his or her reserve capacity c. Ambulating while using an assistive device is known to create a metabolic demand different from that for unassisted ambulation d. Assistive devices are used as transitional aids towards the goal of independent ambulation e. The decreased metabolic demand with the use of assistive devices is at least in part due to the lower speed

a. These devices may be used as permanent functional aids the can be used for home or community ambulation.

This class of lever is found in most open-chain motion of the extremities, e.g. the deltoid acting on the glenohumeral joint: a. Third b. Second c. Fourth d. See-saw e. First

a. Third

In this class of lever, the force is applied at a site intermediate between the fulcrum and the weight to be moved: a. Third class lever b. None of these c. Second class lever d. First class lever

a. Third class lever

In joint where movement takes place about three main axes, all of which pass through the joint's center of rotation, it is said to possess a. Three degree of freedom b. Universal movement c. A degree of freedom d. freedom of movement e. two degree of freedom

a. Three degree of freedom

The Continuing Professional Education (CPE) Council for PT and OT exercise powers and functions that include the following, EXCEPT: a. To present periodic assessment and upgrade criteria for accreditation of CPE providers and CPE programs to the Commission for approval. b. Accept, evaluate and approve applications for exemptions from CPE requirements. c. Periodically monitors the implementation of programs, activities or sources. d. Accept, evaluate and approve applications for accreditation of CPE providers e. Accept, evaluate and approve applications for accreditation of CPE programs, and determine the number of CPE credit units.

a. To present periodic assessment and upgrade criteria for accreditation of CPE providers and CPE programs to the Commission for approval.

The following statements are true of medial collateral ligaments of the knee, EXCEPT: a. To test the ligament, push laterally against the knee and medially against the ankle to attempt a varus stress b. If there is a medial joint line gapping, the ligament is not supporting the knee properly c. A tear on this ligament leads to joint instability, whereas similar defect in the lateral collateral ligament may have little effect either way d. A "chunk" is felt when the tibia and femur close together as a stress on the injured joint is relieved e. To test the ligament, push medially against the knee and laterally against the ankle to attempt a valgus stress

a. To test the ligament, push laterally against the knee and medially against the ankle to attempt a varus stress

An isolated lesion of this nerve results in vertical diplopia and tilts his head to align the eyes: a. Trochlear nerve b. Oculomotor nerve c. Facial nerve d. Abducens nerve

a. Trochlear nerve

A therapist utilizes continuous ultrasound to supply thermal effects to a patient rehabilitating from a lower extremity injury. During the treatment session, the patient suddenly becomes start led and report feeling an electrical shock from the ultrasound machine. The most appropriate therapist action is to: a. Unplug the machine and label-defective, do not use b. Decrease the intensity of the ultrasound c. Modify the cycle duty d. discontinue ultrasound treatment e. reposition head placement location

a. Unplug the machine and label-defective, do not use

A registered professional can be permanently exempt from Continuing Professional Education requirements a. Upon reaching the age of 65 years old b. By becoming a member of the Board of Examiners c. Upon formally requesting for exemption for earning the PRC most outstanding professional award d. Upon completing his/her doctoral degree e. Upon written recommendations from the President and Dean of the University

a. Upon reaching the age of 65 years old

The product of breathing frequency and tidal volume (Vt): a. Ventilation (in liters per minute) b. Stroke volume c. Metabolic rate d. heart rate e. cardiac output

a. Ventilation (in liters per minute)

In the revised category-ratio rating of perceived exertion (RPE), a rating of 7 is described a. Very strong b. Very, very strong c. Somewhat strong d. Weak e. Moderate

a. Very strong

The following statements describe carpal tunnel syndrome, EXCEPT: a. Weakness of the hand. b. The pathophysiology remains unknown although mechanical and vascular factors can play a major role. c. Often seen as the cause of progressive numbness or paresthesia of the fingers in the median nerve distribution d. Numbness or pain that can radiate distally. e. Nocturnal burning pain or hypesthesia.

a. Weakness of the hand.

Energy costs during walking adults with below knee amputations (BKAs) and those without amputation may be generalized as follows, EXCEPT: a. When adults with BKAs walked at the same speed as adults without, their energy costs were approximately 32% greater. b. Energy costs are the same between adults with BKAs and those without when walking at the same speed. c. They have the same energy costs because the person with amputations walked at slower speeds. d. Adults with BKAs and short residual limbs (less than 6% of total body height) has a 40% increase when they perform the same activities. e. Adults with BAKs and long residual limbs (greater than 8% of total body height) had a 10% increase in oxygen consumption compared with adults without.

a. When adults with BKAs walked at the same speed as adults without, their energy costs were approximately 32% greater.

The time a subject can maintain a horizontal, unsupported posture (a measure of mechanical capability and willingness) is a predictor for first-time occurrence of low back pain in men. This test is known as: a. Willliams b. Jorgensen c. McKenzie d. Sorensen e. Biering

a. Willliams

Skeletal muscles are called to perform different series of tasks and accomplish its task in the following manner, EXCEPT: a. With a series of twitches b. May produce a variety of time-varying forces between an impulsive force and titanic contraction. c. May produce an impulsive force d. May produce a steady titanic contraction.

a. With a series of twitches

Compression-side fractures of the inferior side of the femoral neck are characterized by the following statements, EXCEPT: a. With an overt fracture line, bed rest is indicated b. Usually negative radiograph and positive bone scintigraphy initially c. If sclerosis is present without an overt fracture, bed rest is indicated d. Any widening of the fracture line would required stabilization with multiple Kowles pins e. When hip pain is absent at rest, gradual progressive weight bearing and increased activity are begun

a. With an overt fracture line, bed rest is indicated

Area of skin supplied by one dorsal root. a. Erector pili b. Dermatome c. Myotome d. Epidermis e. Dermis

b. Dermatome

Suggested dimension (in feet) for an extremity whirlpool in Hydrotherapy area is a. 8 x 8 b. 5 x 7 c. 7 x 8 d. 10 x 8 e. 8 x 10

c. 7 x 8

Digestion of lactose results in: a. 2 molecules of fructose b. 1 molecule glucose + 1 molecule galactose c. 1 molecule of glucose + 1 molecule fructose d. 2 molecules of glucose e. 1 molecule galactose + 1 molecule fructose

b. 1 molecule glucose + 1 molecule galactose

Composition of the human peripheral nervous system: a. 10 pairs of cranial nerve and 32 pairs of spinal nerves b. 12 pairs of cranial nerve and 31 pairs of spinal nerves c. 11 pairs of cranial nerves and 32 pairs of spinal nerves d. 12 pairs of cranial nerve and 32 pairs of spinal nerves e. 12 pairs of cranial nerve and 33 pairs of spinal nerves

b. 12 pairs of cranial nerve and 31 pairs of spinal nerves

Which of the following meal will be emptied first? a. 300 cal semi solid high protein b. 300 cal liquid high carbohydrate c. 300 cal liquid high protein d. 300 cal solid high protein e. 300 cal solid high carbohydrate

b. 300 cal liquid high carbohydrate

The Act known as the Philippine Physical and Occupational Therapy Law, creating the board of examiners for PT and OTs is known as Republic Act No. a. 8066 b. 5680 c. 5670 d. 8056 e. 5690

b. 5680

A patient using an end-range splinting device in four sessions per day for 30 minutes each session for five days has a "total end-range time" of: a. 150 b. 600 c. 120 d. 20 e. 75

b. 600

The following are true of the spinal (or dorsal root) ganglia, EXCEPT: a. Each spinal nerve possesses one, and the ganglia are found each in intervertebral foramen resting on a pedicle b. A collection of cell bodies whose axons are in the anterior horn of the gray matter of the cord c. Each spinal ganglion's cell body is part of a unipolar, afferent neuron having a T-shaped axon d. All of these

b. A collection of cell bodies whose axons are in the anterior horn of the gray matter of the cord

Sexual dysfunction in spinal cord injury may occur due to the following, EXCEPT: a. Failure of sympathetic input b. A combination of these factors c. Failure of somatic afferent and efferent neurotransmission d. Failure of sympathetic outflows e. Psychological distress stemming from changes in body image and feelings of inadequacy

b. A combination of these factors

If there is a compression of the subclavian artery, one feels a marked diminution or absence of the radial pulse. This test is called a. Swallowing Test b. Adson Test c. Valsalva Test d. Compression Test e. Distraction Test

b. Adson Test

The following statements characterize carpal tunnel syndrome, EXCEPT a. Pain, numbness and tingling of the hands b. Advanced cases may include wasting of the hypothenar eminence and an apparent clumsiness of the hand c. Condition may affect both hands or only the dominant hand d. Symptoms are most often acute while sleeping e. Uncomfortable sensations are usually felt on the first three fingers and the base of the thumb

b. Advanced cases may include wasting of the hypothenar eminence and an apparent clumsiness of the hand

After internal fixation of the hip, the physical therapist must pay special attention to: a. Use of a trochanter roll or properly placed sandbags beside the thigh to avoid extreme external rotation b. All of these c. Patient may not lean forward further than 30 to 40 which would increase hip flexion d. To prevent increased pain and undue strain on the fracture site, adduction, external rotation or acute flexion of the patient's hip must not be allowed e. Use of pillow or abductor triangle to maintain abduction and keep the affected leg to one side of his midline

b. All of these

Fractures, though not usually life threatening, may be caused by: a. Pathologic muscle contractions and pathologic decay b. All of these c. Fatigue or stress d. Direct force e. Torsion force

b. All of these

In closed-packed position: a. Ligament attachments are farthest apart and under tension b. All of these c. The joint difficult to distract and mechanically compressed d. Maximum surface contact occurs

b. All of these

Stimulation of carotid baroreceptors: a. Occurs when blood pressure is elevated, thus stretching the blood vessel walls b. All of these c. Results in decreased heart rate and decreased cardiac contractility d. None of these e. Results in reflex vasodilatation

b. All of these

The following arteries contribute to a rich anastomosis at the base of the brain known as the Circle of Willis: a. The two Vertebral Arteries b. All of these c. The two Internal Carotid Arteries d. None of these

b. All of these

Which of the following is consideration in wheelchair selection? a. For a patient with poor sitting balance, recommend a chair with a solid seat b. All of these c. If your patient has bilateral leg amputations, recommend a chair with wheels set back a few inches d. A wheelchair with several types of hand projections on the handrims is indicated for patients with arm or hand weakness e. None of these

b. All of these

The following statements are true of the shoulder joint, EXCEPT: a. The scapula, which forms the mobile base of the shoulder joint, has minimal passive suspension from the skeleton via the acromioclavicular joint and coracoclavicular ligament b. An implication of the shoulder's structural modification to increase mobility is that it relies on ligaments to provide adequate stability c. The ligaments are few and only provide stability in limited joint ranges of motion d. The joint capsule is thin and lax, allowing 2 to 3 cm. of distraction between articular surfaces e. The articular surfaces of the humeral head and the glenoid fossa of the scapula lack congruity

b. An implication of the shoulder's structural modification to increase mobility is that it relies on ligaments to provide adequate stability

Periarticular connective tissues are designed to withstand tensile stress. If these tissues are deprived of tensile stress by being immobilized in a shortened position, the following changes will occur, EXCEPT: a. Remodeling is halted b. An increase in the amount of reducible crosslink c. There is a decrease in its functional length d. Becomes shorter, weaker, and more stiff e. Exhibit decreased amounts of collagen, glycosaminoglycans and water

b. An increase in the amount of reducible crosslink

Inhibits synthesis of angiotensin Il in the bloodstream and various tissues, and decreases angiotensin ll mediated vasoconstriction and vascular structural changes: a. Alpha-adrenergic blockers b. Angiotensin-converting (ACE) enzyme inhibitors c. Diuretics d. Calcium channel blockers e. Beta-adrenergic blockers

b. Angiotensin-converting (ACE) enzyme inhibitors

This structure tends to prevent the radius from being pulled down out of its socket, and completes a collar in which the radial head revolves: a. Cord b. Annular ligament c. Radial collateral ligament d. Radial notch of the ulna e. Ulnar collateral ligament

b. Annular ligament

The following statement characterize the Radial Nerve, EXCEPT: a. A little father down the arm, it passes obliquely lateralwards to run behind the humerus in its spiral groove, deep to the Triceps b. As it crosses the elbow joint, it supplies the muscles arising from the lateral supracondylar ridge: the Brachioradialis and Extensor Carpi Radialis Brevis c. It is the largest nerve of the upper limb d. Its nerve divides include C5, C6, C7, C8 and e. T1 The nerve divides into a long Superficial Branch and Deep Branch or Posterior Interosseous Nerve.

b. As it crosses the elbow joint, it supplies the muscles arising from the lateral supracondylar ridge: the Brachioradialis and Extensor Carpi Radialis Brevis

The following statements describe the "Carrying Angle", EXCEPT: a. The angularity of the extended and supinated limb disappears on pronation b. As the ulna swings round the trochlear from flexion to extension, the trochlear is forced gradually lateralwards. c. The difference in circumference between the two ends (ulna and trochlear) of the hourglass is responsible for forcing the ulna gradually out of line with the humerus d. The difference in alignment between humerus and ulna is compensated for when the lower end of the radius crosses over the medial side of the ulna e. This difference in alignment between the humerus and ulna amounts to about 15 degrees when the elbow is fully extended

b. As the ulna swings round the trochlear from flexion to extension, the trochlear is forced gradually lateralwards.

These fibers bring neurons of one part of the cortex of a hemisphere into communication a. Projection b. Association c. Assimilation d. Extrapyramidal e. Commissural

b. Association

In the normal upright adult lung, blood flow is greatest at this part of the lung: a. Both at the base and apex b. At the base c. At the apex d. equally distributed throughout e. near the middle

b. At the base

The following statements are true of straight leg raising test, EXCEPT: a. The normal angle between the table and the leg measures approximately 80 degrees b. At the point where the patient experiences pain, lower the leg slightly and then dorsiflex the foot — if the there is no pain, the pain induced is probably due to the sciatic nerve c. Designed to reproduce back and leg pain d. If there is a positive reaction to the straight leg raising test and the foot dorsiflexion maneuver, the pain may be either in the lumbar spine or along the course of the sciatic nerve e. The foot is lifted upward supporting the calcaneus, and with the knee remaining straight, raise the leg to the point of discomfort or pain

b. At the point where the patient experiences pain, lower the leg slightly and then dorsiflex the foot — if the there is no pain, the pain induced is probably due to the sciatic nerve

When measuring hip abduction the stationary arm of the goniometer should be positioned a. Along the midline of the linea alba b. Between anterior and superior iliac spines. c. Along a line from the crest of the ilium, femur and greater trochanter d. Parallel to the anterior aspect of the femur e. Along the midline of the tibia

b. Between anterior and superior iliac spines.

The entire flexor pronator can be surgically removed from its common origin and transferred proximally onto the humerus to substitute for a weak or absent: a. Triceps b. Biceps muscle c. Brachioradialis d. Coracobrachialis e. Brachialis

b. Biceps muscle

The muscles of mastication are composed of the following, EXCEPT: a. Temporalis b. Biceps muscle c. Brachioradialis d. Coracobrachialis e. Brachialis

b. Biceps muscle

Stimulation of thermoreceptors in the anterior hypothalamus results in: a. Vasodilatation b. Both of these c. Sweating d. None of these

b. Both of these

Electrical stimulation affects sensory and motor nerves in the following manner, EXCEPT: a. Varying the waveform has not meaningful influence on their responses. b. Both respond in a seemingly identical way to transcutaneous stimulation. c. Sensory excitation always precede motor excitation irrespective of waveform d. Motor nerve thresholds are higher than sensory e. Motor excitation can precede sensory excitation depending on site of stimulation

b. Both respond in a seemingly identical way to transcutaneous stimulation.

Muscles will develop strength very rapidly even if the contractions are performed only a few times each day and if they contract at or near their maximal force of contraction. a. Both statements are false b. Both statements are true c. First statement is false, second statement is true d. First statement is true, second statement is false

b. Both statements are true

This elbow flexor is innervated by the radial nerve and attaches proximally to a ridge on the humerus above the lateral epicondyle: a. None of these b. Brachioradialis c. Biceps d. Brachialis

b. Brachioradialis

Following statements describe fibromyalgia, EXCEPT: a. A nonarticular rheumatic disease of unknown origin characterized by tenderness at 11 or more of the 18 tender points sites b. Cardiovascular training offers no benefit in reducing the symptoms c. There may be patient-reported joint swelling without objective findings of swelling d. The tender points are primarily localized to the musculotendinous junctions e. Generalized fatigue, chronic headache and sleep disturbance may be present

b. Cardiovascular training offers no benefit in reducing the symptoms

Hearing impairment resulting from damage to the inner ear or neural fibers of the eight cranial nerve: a. Peripheral b. Conductive c. Mixed d. Facial e. Abducens

b. Conductive

Plantar-flexion contractures can interfere with the performance of functional tasks through the following, EXCEPT: a. Soft tissues passively limit dorsiflexion to the muscle moment developed about the ankle during the stance phase. b. Decrease the plantar flexor moment when the ankle is in dorsiflexed positions. c. There is knee hyperextension during the stance phase d. There is decreased ankle dorsiflexion during the swing phase e. There is decreased peak hip extension in late stance phase

b. Decrease the plantar flexor moment when the ankle is in dorsiflexed positions.

Transaction of the pituitary stalk will produce a. Increased LH ant b. Decreased TSH ant c. Decreased prolactin d. Increased FSH e. Increased ACTH

b. Decreased TSH ant

Injury to the right superior gluteal nerve will result in this gait deviation: a. Increased pressure in heel strike b. Decreased stride width c. Lateral flexion of the trunk to the contralateral side d. Increased swing phase of reference leg e. Increased duration of weight bearing as the reference extremity

b. Decreased stride width

A therapist completes a cognitive function test on a patient status post stoke. As part of the test, the therapist examines the patient's abstract ability. Which of the following tasks would be the most appropriate? a. Verbalize a position statement b. Discuss how two objects are similar c. Repetition of a series of letters d. repetition of a series of numbers e. a figure from a picture

b. Discuss how two objects are similar

A therapist reviews the medical record of a patient with cancer. The medical record includes a number of advanced directives including a "do not resuscitate" order. The therapist is very upset to learn this information and feels the patient is giving up hope. The most appropriate actions is a. Continue with the patient's established care plan b. Discuss your concerns with the patient's physician c. Explain the benefits of positive thinking to the patient d. Inform the patient about other possible treatment alternatives e. Explain your concerns to the patient's family

b. Discuss your concerns with the patient's physician

Disinfection is a process distinct from sterilization. The following statements are true, EXCEPT: a. Something that is sterile is free of any microbial life. b. Disinfection is a process that results in the complete removal of all microbial life from an object. c. Sterilization may be accomplished through autoclaving of irradiation by x-rays or gamma rays d. Disinfection tends to kill vegetative microorganisms or inhibit their growth e. Most disinfectants are chemicals, generally used on inanimate objects or surfaces rather than on living tissues.

b. Disinfection is a process that results in the complete removal of all microbial life from an object.

This test describes the ability of the abdominal muscles to maintain the pelvis in a position of posterior pelvic tilt as the fully extended legs are slowly lowered to the table from a position of 90 degrees of hip flexion. a. Thomas test b. Double leg lowering test c. None of these d. Straight leg raising test e. Modified Thomas test

b. Double leg lowering test

The combined action of these muscles is vertebral extension, are continuous from the sacrum to the occiput: a. None of these b. Erector Spinae or Sacrospinalis c. Transversopinal d. Quadratus Lumborum

b. Erector Spinae or Sacrospinalis

Following statements are true of lower motor neuron lesions, EXCEPT: a. Muscle atrophy may begin to show within a few weeks b. Even with the loss of sensory feedback to the nervous system coordinated movements are still properly accomplished c. The muscle loses both its voluntary and reflex response d. Hypotonia is observed as well as absence of muscle stretch reflex

b. Even with the loss of sensory feedback to the nervous system coordinated movements are still properly accomplished

Components of subtalar joint pronation when the foot is bearing weight: a. Excessive calcaneal eversion, talar abduction and talar dorsiflexion b. Excessive calcaneal eversion, talar adduction and talar plantar flexion c. Moderate calcaneal inversion and talar dorsiflexion d. Excessive calcaneal inversion, talar adduction, and talar plantar flexion e. Talar adduction and talar dorsiflexion and moderate calcaneal inversion

b. Excessive calcaneal eversion, talar adduction and talar plantar flexion

A form of Validity that concerns the extent to which a test of measure appears to measure what it suppose to measure, and actually based on the personal opinion of those either taking or giving a test. a. Content Validity b. Face Validity c. Direct Validity d. Criterion-Related Validity e. Construct Validity

b. Face Validity

Fast twitch fibers differ from slow twitch fibers, EXCEPT: a. Number of capillaries per mass of fibers in greater in slow twitch fibers b. Fast twitch fibers are organized for endurance c. Fast twitch fibers can achieve double the maximal power d. Fast twitch fibers are twice in diameter

b. Fast twitch fibers are organized for endurance

These muscles are said to place the hand in the "writing" position (metacarpophalangeal joints flexed, interphalangeal joints extended): a. Flexor Digitorum Superficialis b. Four lumbricals c. Palmar interossei d. Dorsal interossei

b. Four lumbricals

Stimulation of mass movements after ingestion of a normal meal is called: a. lleal break b. Gastrocolic reflex c. Enterogastric reflex d. gastrosphincteric reflex e. retrosphincteric reflex

b. Gastrocolic reflex

True hip pain in rheumatoid involvement of the hip produces pain on the a. Lateral aspect of the high b. Groin c. On the acetabular area d. medial aspect of the thigh e. posterior aspect of the thigh

b. Groin

Vertical deformities of the toes may be caused by the following, EXCEPT: a. When wearing footwear regardless of fit b. Hallux valgus c. Inflammatory arthritis d. phalange fracture e. peroneal nerve palsy

b. Hallux valgus

A therapist observes a video on the biomechanics of normal gait. The therapist notes that the subject's knee remains flexed during all of the components of stance, EXCEPT: a. Midstance b. Heel strike c. Fool flat d. endstance e. toe off

b. Heel strike

Hypocoagulopathy can lead to a. Increased heart workload b. Hemorrhage and edema c. Embolus formation d. thrombus formation e. increased breathing

b. Hemorrhage and edema

The following characteristics describe Type IIA muscle fibers, EXCEPT: a. Contained in motor units characterized by a high firing frequency or discharge rate b. High threshold for activation and slow-twitch contraction. c. Have cytological properties that fall between type | and type IIB fiber d. Relatively fatigue resistant e. With intermediate levels of glycolytic and oxidative enzyme activity

b. High threshold for activation and slow-twitch contraction.

Gait in the hemiplegic is described as follows, EXCEPT: a. The rectus femoris resists knee flexion in the swing phase b. Hip hiking is related to ipsilateral trunk leaning and abduction of the contralateral hip c. Toe-off is delayed on the affected side d. The hip and knee flexes to clear the foot which the ankle is plantar flexed through the swing phase e. Forward momentum of the thigh results in backward movement of the pelvis

b. Hip hiking is related to ipsilateral trunk leaning and abduction of the contralateral hip

Diffusion occurs faster if a. The temperature is lower. b. The partition coefficient is higher c. The partition coefficient is higher and the molecular size is bigger d. The molecular size is bigger e. The molecular size is bigger and the temperature is higher

b. The partition coefficient is higher

Allograft rejection is a result of a. Formation of antibodies and sensitized lymphocytes, either or both of which may destroy the antigen, a property of acquired immunity b. Histocompatibility complexes c. That function to which protects an individual from toxins that might cause tissue damage d. Destruction of the antigen by phagocytosis, a property of innate immunity e. Provoked immunity

b. Histocompatibility complexes

An individual with rheumatoid arthritis that is largely or wholly incapacitated, bedridden or using a wheelchair, permitting little or no self-care is classified as Class: a. V b. IV c. I d. III e. II

b. IV

Lesions in the primary motor region result in the following statements, EXCEPT a. A few weeks after the immediate body response, enhanced muscle stretch reflexes and Babinski's sign is observed b. Immediate paresis of the ipsilateral musculature with increased resistance to passive manipulation c. Immediate paresis of the contralateral musculature with hypotonia and diminished muscle stretch reflexes d. A few weeks after the immediate body response, partial recovery of muscle strength and development of spasticity of the affected musculature appears

b. Immediate paresis of the ipsilateral musculature with increased resistance to passive manipulation

The following statements describe fatigue, EXCEPT: a. Impairment of transmission at the myoneural junction b. Increase in frequency of activation of each functioning motor unit c. Impairment of excitation contraction coupling d. Reduction of functioning motor units

b. Increase in frequency of activation of each functioning motor unit

Neuromuscular electrical stimulation has been used for the following, EXCEPT: a. Facilitate motor control b. Increase strength c. Temporarily reduce hyperreflexia d. Improve range of motion e. Muscle reeducation

b. Increase strength

In an open-packed or loose packed position, EXCEPT: a. The ligamentous and capsular structures are slack b. Increased joint friction allow spinning and sliding c. Joint surface may be distracted several millimeters d. The ovoid joint surfaces are incongruent e. Allows spinning, rolling, and sliding, thereby decreasing joint friction.

b. Increased joint friction allow spinning and sliding

Receives the weight of the body when one sits up straight: a. Inferior ramus b. Ischial tuberosity c. Lesser sciatic notch d. Spine of ischium

b. Ischial tuberosity

When the rate of movement is constant, this contraction occurs: a. Eccentric b. Isokinetic c. Concentric d. Isometric

b. Isokinetic

Stabilize joints and produces force with no gross change in the joint angle: a. Concentric b. Isometric c. Isotonic d. Isokinetic e. Eccentric

b. Isometric

This type of contraction occurs when muscular tension equals the opposing force a. Relaxed b. Isometric c. Isotonic d. Concentric e. Eccentric

b. Isometric

The following statements characterize Tensor Fasciae Latae, EXCEPT: a. It nerve supply is by the superior gluteal nerve, a branch of the sacral plexus b. It is inserted into the iliotibial tract of the fascia latae with Gluteus Medius c. It can help abduct, medially rotate and flex the hip joint d. Arises from the outer surface of the ilium just behind the anterior superior spine and runs vertically downwards e. Its important use is to brace the knee so that in walking, the joint can take the weight without "buckling" while the other foot is off the ground and the body is swinging forwards

b. It is inserted into the iliotibial tract of the fascia latae with Gluteus Medius

Which of the following statements is true of the Levator Palpebrae Superioris? a. None of these b. It is the highest muscles of the orbital cavity which raises the upper eyelid c. All of these d. The margins of the muscle in front are fixed to the medial and lateral sides of the orbital cavity to safeguard against the muscle pulling the upper lid into the orbital cavity.

b. It is the highest muscles of the orbital cavity which raises the upper eyelid

Cranial nerve integrity involves several components, Tests and measures are used to localize dysfunction in the brain stem and cranial nerves, and are appropriate in the presence of the following impairments, EXCEPT: a. Aerobic capacity and endurance b. Joint integrity and mobility c. Gait, locomotion, and balance d. neuromotor development and sensory integration e. muscle performance

b. Joint integrity and mobility

The most abundant cation inside the living cell is a. Na+ b. K+ c. Mg++ d. Cl- e. Ca++

b. K+

Feedback information about the kinematic or kinetic components of the movement being attempted by the patient. a. Result Oriented b. Knowledge of Performance c. Knowledge of Function d. Knowledge of Results e. Performance Oriented

b. Knowledge of Performance

Normally, the movement of the contents of the gastrointestinal tract is slowest along this segment. a. Mouth b. Large intestine c. Esophagus d. Small intestine e. Stomach

b. Large intestine

The following statements describe lesions in lumbosacral levels, EXCEPT: a. b. The spinal cord ends at the L-1 to L-2 vertebral level in adults b. Lesions above the L-1 vertebral level which interrupt the descending spinal cord pathways yet spare the sacral components c. The sacral components are not spared in lesions above the L-1 vertebral level d. The vertebral level of injury does not always correlate with the spinal segment level of injury e. In adults, injuries at or below the L-1 vertebral level that damage the S2 to parenchyma will damage the lower motor neurons and parasympathetic center located in the S-2 to S-4 spinal cord segments.

b. Lesions above the L-1 vertebral level which interrupt the descending spinal cord pathways yet spare the sacral components

The Median Frequency during a sustained fatiguing isometric contractions shifts or compresses toward a. Increased frequencies b. Lower frequencies over time c. Middle frequencies over time d. alternating frequencies e. higher frequencies over time.

b. Lower frequencies over time

The agonist maybe described as follows, EXCEPT: a. Contracts actively to produce a concentric, isometric or eccentric contraction. b. The principal muscle maintaining a posture c. The principal muscle producing a joint motion d. Contracts actively to produce a concentric and eccentric contraction. e. Prime mover

b. The principal muscle maintaining a posture

The following statements apply to Buck's extension traction, EXCEPT: a. A type of skin traction applied to the lower extremities b. May be used to immobilize the forearm, or to treat shoulder problems c. To decrease edema and promote venous blood return to the heart, an embolism stocking is first applied d. Must be removed at least once every 8 hours from a period of 1 hour to avoid skin complication e. Straps are tightened as you proceed up the leg; when the finger won't fit under the strap, tightness is about right

b. May be used to immobilize the forearm, or to treat shoulder problems

The injury causes paralysis of the Pronator teres and Quadratus, disabling movement such as holding a glass: a. Radial nerve injury above the elbow b. Median nerve injury above the elbow c. Ulnar nerve injury at wrist level d. Ulnar nerve injury above the elbow e. Median nerve injury at wrist level

b. Median nerve injury above the elbow

When one shifts to an upright from supine position: a. The lungs receive a greater portion of the cardiac output b. More blood shifts to the venous portion of the circulation c. The heart receives a lesser portion of the cardiac throughout d. The heart receives a greater portion of the cardiac output e. More blood shifts to the arterial portion of the circulation

b. More blood shifts to the venous portion of the circulation

This nerve provides innervations to the superior, inferior and medial rectus, and the inferior oblique muscle of the eye. a. Trigeminal b. Oculomotor c. Optic d. Abducens e. Trochlear

b. Oculomotor

Muscular hypotonia is seen in the following conditions, EXCEPT: a. Poliomyelitis b. Parkinson's syndrome c. Peripheral nerve injury d. Cerebellar dysfunction

b. Parkinson's syndrome

Gait patterns between young (20-40 y.o.) and old (60-80 y.o) people differ in their gait performance as follows, EXCEPT: a. Ankle dorsiflexion is slightly increased in older, while ankle plantar flexion was decreased in older people. b. Peak hip flexion exhibit a slightly increase in the older persons. c. Peak knee extension is significantly less in the older persons. d. Stride length is significantly greater in the younger persons. e. Stride time is shorter for the young than the old

b. Peak hip flexion exhibit a slightly increase in the older persons.

A set of attributes that people have or achieve that relates to the ability to perform physical activity: a. Exercise b. Physical fitness c. Physical activity d. Work out e. Aerobics

b. Physical fitness

After transverse rectus abdominals myocutaneous flap procedure, the following techniques may be used to reduce stress on the sutures of the abdominal wound closure EXCEPT: a. Teach log-rolling techniques for moving safely in bed without disrupting the sutures b. Positioning the bed at a 90-degree angle, patient lying on the uninvolved side in a fetal position. c. Lifting and sit-ups are not permitted until a 6 weeks after surgery d. Exercises designed for co-contraction of the oblique, transversus abdominis and multifidus muscles for optimal trunk strengthening e. Avoid Valsalva maneuver by exhaling during physical exertion

b. Positioning the bed at a 90-degree angle, patient lying on the uninvolved side in a fetal position.

In this test, the patient is asked to place the thumb inside his closed fist. If passive or active ulnar deviation of the wrist produces pain over the styloid process of the radius, this indicates a: a. Peripheral neuropathy b. Positive Finkelstein's test c. Positive Hoffman sign d. negative disdiadokokinesia e. negative Finkelstein's test

b. Positive Finkelstein's test

This structure resist the tendency of the femur to slide forwards off the plateau like tibial surface in actions such as in jumping with knees bent: a. Lateral collateral ligament b. Posterior cruciate ligament c. Medial and lateral menisci d. Anterior cruciate ligament e. Medial collateral ligament

b. Posterior cruciate ligament

A manager develops a policy on physical and occupational therapy utilization of continuing education resources. Which of the following would be most appropriate action to enhance the quality of patient care? a. b. 154. First come first served basis, i. e., continuing education is offered to therapists who reserved first b. Prioritize requests for continuing education resources based on established patient care standards c. Offer continuing education resources to senior therapist is relation to their years of experience. d. Establish a committee to review requests for continuing education resources e. Divide the continuing education resources evenly among therapy staff.

b. Prioritize requests for continuing education resources based on established patient care standards

9. The most appropriate position to test the strength of a patient's middle trapezius is a. Sitting unsupported b. Prone c. Supine d. Sitting supported e. Side lying

b. Prone

The segment of nephron which is permeable to water is the a. Thin descending limb of the loop of Henle b. Proximal tubule c. Thin ascending limb of the loop of Henle d. Distal convoluted tubules e. Thick ascending limb of the loop of Henle

b. Proximal tubule

The angle formed by the tendons of the quadriceps and ligamentum patella with the center of the patella: a. Patellar angle b. Q angle c. Genu valgum d. Genu recurvatum e. Genu Varum

b. Q angle

The most important substance regulating blood pressure that is produced by the kidney is a. Vasopressin b. Renin c. Atrial natriuretic hormone d. Prostaglandin e. Progesterone

b. Renin

The following statement characterize De Quervain's disease, EXCEPT: a. Combinations of hand twisting and forceful gripping can cause the disease b. Repetitive friction accounts for the absorb thickening of the fibrous sheath and resultant contraction of the tendons c. Attributed to excessive friction between two thumb tendons and their common sheath d. Affects the tendons on the side of the wrist and at the base of the thumb e. The tendons affected are connected to the muscles on the back of the forearm that contract to pull the thumb back and away from the hand.

b. Repetitive friction accounts for the absorb thickening of the fibrous sheath and resultant contraction of the tendons

The following statements apply to myocardial ischemia and infarction, EXCEPT: a. Characteristic symptoms of pain associated with angina pectoris is caused by an imbalance between myocardial oxygen supply and demand b. Resolving thrombus formation in the coronary arteries cannot control factors leading to coronary occlusion c. Caused primarily by coronary artery atherosclerosis that decreases the ability of the coronary arteries to supply adequate oxygen to meet the demands of the myocardium d. Progressive atherosclerosis also leads to the development of coronary artery thrombosis resulting in vessel before occlusion and MI e. Drugs are often used to treat the symptoms of ischemic heart disease and restore myocardial oxygen balance before additional damage occurs to the heart

b. Resolving thrombus formation in the coronary arteries cannot control factors leading to coronary occlusion

These carpal bones form the radiocarpal joint by their biconcave distal end of the radius and the biconcave proximal articulating surfaces. a. Scaphoid and capitate b. Scaphoid and lunate c. Capitate and lunate d. Lunate and capitate e. Scaphoid and trapezoid

b. Scaphoid and lunate

Almost the whole of the broad sacral plexus narrows down to form a huge branch called: a. Saphenous nerve b. Sciatic nerve c. Femoral nerve d. Genitofemoral nerve e. Obturator nerve

b. Sciatic nerve

The following statements apply to the lateral femoral cutaneous nerve, EXCEPT: a. There is no atrophy and no motor or reflex change b. Sensory and motor function is mediated by this nerve c. Some sensory loss to pain and touch is typical d. More apt to occur with metabolic disorders e. Entrapment of this where it passes under the inguinal ligament medial to the anterior superior iliac spine results in a syndrome of dysesthesia and pain along the lateral thigh called meralgia paresthetica

b. Sensory and motor function is mediated by this nerve

This goniometric measurement should NOT be tested with the patient in supine? a. Hip abduction b. Shoulder extension c. Knee flexion d. hip flexion e. elbow extension

b. Shoulder extension

Theoretical relationships are established between the primary concept to be measured and one or more other concepts in this type of validity. a. Pessimistic outlook over the years b. Significant loss of fat in the muscle fibers of the face c. Denervated facial nerve d. None of these e. All of these

b. Significant loss of fat in the muscle fibers of the face

Oscillations of resting membrane potentials of the Gl smooth muscle not associated with muscle contraction is known as: a. Fast waves b. Slow waves c. Spikes d. action potentials e. tone

b. Slow waves

The following are examples of deep somatic sensation, EXCEPT: a. Deep pain b. Strong smell c. Proprioception d. Vibration sense e. Deep pressure

b. Strong smell

Optic radiation lesions could result in the following visual field defects, EXCEPT: a. The more posterior the lesion, the more nearly do the visual field defects in the two eyes resemble each other b. Temporal lobe lesions result in an ipsilateral homonymous superior quadrantinopsia c. Parietal lobe lesions is a contralateral homonymous, inferior quadrantinopsia d. Interruption of the entire radiation gives complete loss of vision to the opposite side, homonymous hemianopsia

b. Temporal lobe lesions result in an ipsilateral homonymous superior quadrantinopsia

A general term for a repetitive-induced tendon injury involving the synovial sheath: a. Epicondylitis b. Tenosynovitis c. Cystitis d. Tendinitis e. Stenosis

b. Tenosynovitis

When applied, second class levers can be levers a. Of stability and speed b. That work at a mechanical advantage c. That operate at a mechanical disadvantage but on of speed d. That operate at a mechanical disadvantage but one of stability e. Of stability only

b. That work at a mechanical advantage

Sudden stretch of a skeletal muscle results in a reflex contraction of that muscle mediated by a simple and usually monosynaptic reflex arc. The following statements are true of the reflex arc, EXCEPT: a. Intramedullary fibers synapse on the motor neurons in the neurons in the anterior horn of the spinal cord or on motor nuclei in the lower brain stem. b. The afferent side of the arc begins with muscle stretch receptors whose cell bodies are n the ventral root ganglia c. Enhanced response in the reflex arc may mean disease of the cord, brain stem or hemispheres and is a basic characteristics of the spastic state d. The stretch reflex will be exaggerated when normal function of the pyramidal tract above the lower motor neuron is chronically suppressed or destroyed. e. The efferent side of the arc is the motor neuron with its axon and terminal structure which innervate the muscle

b. The afferent side of the arc begins with muscle stretch receptors whose cell bodies are n the ventral root ganglia

The following statements characterize the Atlanto-Axial joints, EXCEPT: a. A pair of alar ligaments serves to limit the rotation of the head b. The atlas and axis are united by three joints, two paired and one medially placed c. The gliding movement occurs on the axis d. The paired lateral atlanto-axial joints are plane joints and lie directly below the paired atlanto-occipital joints e. When the "no" movement occurs, the atlas and axis move on the skull as a unit and the median atlanto-axial joint rotates in a collar formed by the anterior arch of the atlas and the transverse ligament

b. The atlas and axis are united by three joints, two paired and one medially placed

Functional capacity evaluation are often used to determine the following, EXCEPT: a. A person's readiness to return to work b. The degree of the person's disability in order to collect disability pay c. The need for further rehabilitation d. Physical capabilities and work tolerance e. The need for job modification

b. The degree of the person's disability in order to collect disability pay

The following statements describe Closed Kinematic Chain, EXCEPT: a. The distal segment is fixed and proximal parts move b. The distal segment of the chain moves in space. c. Stair climbing is an alternation closed-chain motion during the support phase of the extremity d. Occurs in the upper extremity when performing chip up. e. All segments are required to move in the movement of one segment

b. The distal segment of the chain moves in space.

In polyneuropathy, lesions often occur bilaterally, the effects are usually more prominent in the proximal than in distal parts of the extremities. a. The first statement is false, the second statement is true b. The first statement is true, the second statement is false c. Both statements are true d. Both statements are false

b. The first statement is true, the second statement is false

Soft tissue palpation of the lumbar spine may present the following observation, EXCEPT: a. Activity flexing the hip when there is psoas abscess results in increased pain in the inguinal area indicating hip joint pathology b. The sacral triangle is a common area of pain due to low back strains or the avulsion of a tendon from the anterior iliac spines. c. Fibro fatty nodules that may be lodged under the lip of the iliac crest's posterior portion may be tender to palpation and cause localized low back pain d. With the hip flexed, the sciatic nerve may be palpated midpoint between the ischial tuberosity and the greater tubercle and can be tender to palpation when a herniated disc is present e. If either the supraspinous or interspinous ligaments are ruptured, the area maybe tender and a defeat may be palpable between the spinous processes.

b. The sacral triangle is a common area of pain due to low back strains or the avulsion of a tendon from the anterior iliac spines.

These neural difference provide part of the basis for the differences in motor behavior that are observed following perinatal brain damage as compared with those found after adult-onset brain damage, EXCEPT: a. Myelinization is not yet complete b. There is an abundance of neurons projecting from the cerebral cortex to the brain stem and spinal cord c. Neurons within the brain has not completely differentiated d. Some corticofugal projections project to inappropriate targets e. There is an overabundance of neurons in certain areas such as the spinal cord

b. There is an abundance of neurons projecting from the cerebral cortex to the brain stem and spinal cord

The following statements characterize lesions of the posterior roots, EXCEPT: a. Each posterior root nerve possesses a posterior root ganglion, the cells of which give rise to peripheral and central nerve fibers. b. There is no loss of associated muscle reflex c. Injury usually results in pain and paresthesia that occur in the distribution of the affected roots. d. Frequent of sensation in a dermatomal distribution is apparent.

b. There is no loss of associated muscle reflex

Which crutch-walking technique would you choose for patients with a walking cast, degenerative joint disease or new leg prosthesis; or for patient who has had leg surgery? a. Swing-through three-point gait b. Three-point-and-one-gait (partial-weight-bearing) c. Three-point gait (non-weight bearing) d. Four-point gait e. Two-point gait

b. Three-point-and-one-gait (partial-weight-bearing)

The following are characteristics of below-knee amputees, EXCEPT: a. Positioning the body weight more posterior with respect to the knee would increase the external knee flexion moment, thus placing a higher demand on the quadriceps femoris musculature b. To compensate for decreased tibial mobility, progression is augmented through a backward trunk lean c. There may be decreased tibial speed, which may be attributed to the lack of normal ankle mechanics because of the cushioned heel keeps body weight on the heel d. Increasing stride length during fast walking is more dependent on torque of the amputated limb knee extensors e. There is prolonged hip extensor activity, more than twice the duration and intensity of normal

b. To compensate for decreased tibial mobility, progression is augmented through a backward trunk lean

Failure to integrate this reflex could explain a child's inability to flex their neck while in a supine position: a. Abdominal reflex b. Tonic labyrinthine c. Moro d. symmetrical tonic neck e. asymmetrical tonic neck

b. Tonic labyrinthine

An involuntary trembling and usually used to describe rhythmic alternating movements of a part. May appear largely at rest or be manifest movement: a. Chorea b. Tremor c. Hyperkinesia d. Myoclonus e. Athetosis

b. Tremor

A patient points to the lateral aspect of the thigh as his source of pain. This may indicate that underlying problem may be due to a. Rheumatoid arthritis b. Trochanteric bursitis c. Gluteal bursitis d. sacroiliac stenosis e. lumbar spinitis

b. Trochanteric bursitis

Of the body water compartment, the largest is occupied by the a. Transcellular fluid b. Trochanteric bursitis c. Gluteal bursitis d. Spinal cord e. Interstitial fluid

b. Trochanteric bursitis

Motor innervations mainly responsible for phonation: a. Hypoglossal b. Vagus c. Phrenic d. Trigeminal e. Accessory

b. Vagus

Cerebrospinal fluid fill these cavities within the brain: a. Cistern b. Ventricles c. Subarachnoid space d. Meninges e. Sinus

b. Ventricles

Forearm supination is measured with the moving arm of the goniometer placed on this side of the hand. a. Dorsal b. Volar c. Lateral d. Medial

b. Volar

This test requires the patient to lie supine and raise his uninvolved leg if there is back and sciatic pain on the involved side, there may be evidence of space occupying lesion in the lumbar area: a. Milgram test b. Well Leg Straight Leg Raising Test c. Kernig Test d. Straight Leg Raising Test e. Hoover Test

b. Well Leg Straight Leg Raising Test

In Erb-Duchenne Palsy, which of the following movements are not affected: a. Elbow flexion b. Wrist extension c. Forearm supination d. Shoulder flexion

b. Wrist extension

If a 40 kilogram force is applied over an area of four square centimeters, the pressure (in kilograms per square centimeter) is equal to a. 20 b. 5 c. 10 d. 160 e. 80

c. 10

Which of the following is true of the coracoclavicular ligament? a. The coracoclavicular ligament travel altogether in all of coracoclavicular joint movements of any considerable extent b. Due to the fibers oblique direction, the ligament transmits forces applied to the scapula at the shoulder region, to the medial 2/3 of the clavicle c. All of these d. A fibrous joint that allows some independent movement between the clavicle and coracoids process

c. All of these

The following statements apply in combined posterolateral and anteromedial rotator instability, EXCEPT: a. Adduction and abduction stress test results are positive at 30 degrees of flexion b. Result of the anterior drawer test is positive with the tibia in neutral position c. Anterior drawer test causes the tibia to rotate externally and back d. Occurs with tear of he medial and lateral compartment ligaments e. Results of the anterior drawer test is negative with tibia in internal rotation

c. Anterior drawer test causes the tibia to rotate externally and back

The most important factor giving rise to a concentrated urine is the presence of a. Digital convoluted tubule b. Proximal tubule c. Anti-diuretic hormone d. Well develop zonula occludes at the collecting tubule e. Non-reabsorbable solute in the lumen of the connecting tubule

c. Anti-diuretic hormone

Thin, strong sheet of fibrous tissue that occasionally attaches flattened muscles. a. Raphe b. Tendons c. Aponeurosis d. Retinaculae

c. Aponeurosis

In patients with movement dysfunction, feedback on a patient's movement performance is an integral approach to rehabilitation, and may be provided in the following manner, EXCEPT: a. Via augmented visual feedback b. Center of pressure information during balance activities c. As the patient attempts to perform a movement, feedback may be provided initially d. Verbally, through the tactile cues transmitted by the therapist's hands

c. As the patient attempts to perform a movement, feedback may be provided initially

Characterized by an inability to sustain a body part or parts in one position, often the distal limbs, and the movements are slow and fluid: a. Dystonia b. Chorea c. Athetosis d. Ballisimus e. Spasticity

c. Athetosis

The Lateral Longitudinal Arch consist of the following, bones: a. Calcaneum, Cuboid, Fourth and Fifth Metatarsal Bones b. Bases of the Metatarsal Bones and the Cuboid and the Three Cuneiform Bones c. Calcaneum, Talus, Navicular Bone, Three Cuneiform Bones and the first Three Metatarsal Bones d. None of these

c. Athetosis

The surfaces of this joint are nearly flat and circular. Gliding movement occur, the one that shakes the head "No" movement: a. Axial-Occipital joints b. Atlanto-occipital joint c. Atlanto-Axial joints d. none of these

c. Atlanto-Axial joints

As gliding occurs, the "No" movement takes place on the surface of these joints that are nearly flat and circular. a.. Axial b. Atlanto-occipital c. Atlanto-axial d. Axial-occipital e. Occipital

c. Atlanto-axial

Skeletal muscles store just enough ATP to provide chemical energy for: a. At lest 1 min. continuous contractions b. At least 15 sec. continuous contractions c. Atleast 30 sec. continuous contractions d. Minimum five strong muscles contractions e. At most three strong muscle contractions

c. Atleast 30 sec. continuous contractions

This uncovering of the reactive site in the actin filament will result in a. Flexion of myosin cross bridge b. Release of calcium from SR site c. Attraction and lengthening of myosin cross bridge d. deformation of troponin e. sliding of actin and myosin

c. Attraction and lengthening of myosin cross bridge

The awareness referred to as Kinesthesia is described as follows, EXCEPT: a. Impulses are transmitted predominantly over group Il afferent fibers b. Awareness of joint motion c. Awareness of static motion d. Impulses are sent to the spinal cord, cerebellum and sensory nuclei e. Kinesthetic signals are generated in response to movements and tension within the tendons

c. Awareness of static motion

This vitamin is best absorbed in the ileum: a. E b. K c. B12 d. C e. A

c. B12

This is formed when synovial fluid extends into the popliteal space and becomes exposed in a membrane. a. Deep vein thrombosis b. Lateral meniscitis c. Baker's cyst d. Thrombophlebitis e. Inflammation of the posterior cruciate ligament

c. Baker's cyst

Decreases catecholamine stimulation of the heart; may also cause a general decrease in systemic sympathetic activity: a. Calcium channel blockers b. Diuretics c. Beta-adrenergic blockers d. Angiotensin-converting (ACE) enzyme inhibitors e. Alpha-adrenergic blockers

c. Beta-adrenergic blockers

A severed peripheral nerve has some capacity to repair itself, and axis cylinders sprout from nerve endings at a rate of 1 to 2 mm per day, with some going astray. Chance determines whether connections can be reestablished and function restored. a. The first statement is true, the second statement is false b. Both statements are false c. Both statements are true d. The first statement is false, the second statement is true

c. Both statements are true

Small unmyelinated and slow conducting nerve fibers are of type a. A-gamma b. A-delta c. C d.D e.B

c. C

The following are differences between closed (CKC) and open kinematic (CKC) chain exercises, EXCEPT: a. OKC are typically non-weight bearing, with movement occurring at a single joint b. In CKC, movement at several joints is required to complete the movement c. CKC are typically weight bearing, an example would be a knee extension exercise, performed in a sitting position with resistance applied to the distal segment d. In CKC, the distal segment is usually fixed to a supporting surface, and the resistance may be applied both proximally and distally. e. In OKC, the distal segment is free to move, and the resistance is usually applied to the distal segment

c. CKC are typically weight bearing, an example would be a knee extension exercise, performed in a sitting position with resistance applied to the distal segment

The following describe clotting abnormalities, EXCEPT: a. Presence of clots usually rules out movement in the affected limb b. Arterial occlusion and thromboemboli can become lodged in arteries supplying vital organs c. Can be caused by mobility and an increase in blood cells d. Patients are prone to emboli and bleeding e. Can originated in the deep veins of the legs

c. Can be caused by mobility and an increase in blood cells

Learning the trunk forward when standing from a sitting position is better accomplished because: a. Forward shifting of the line of gravity b. Increased posterior torque c. Center of gravity is shifted forward d. Increased base of support

c. Center of gravity is shifted forward

Congenital muscular torticollis is a musculoskeletal anomaly with the following characteristics, EXCEPT: a. Children referred to PT before one year of age have better outcomes than those referred later. b. Conservative treatment approach is a stretching program to lengthen the involved muscle c. Clinical features include a head tilt away from the side of the shortened muscle and head rotation toward the ipsilateral side. d. Restricted neck range of motion secondary to a shortened sternocleidomastoid e. A surgical treatment approach is lengthening of the involved muscle

c. Clinical features include a head tilt away from the side of the shortened muscle and head rotation toward the ipsilateral side.

A pathway of communication between the two hemispheres is created as these fibers a. Transhemisphere b. Projection c. Commissural d. Superior longitudinal bundle e. Association

c. Commissural

Removing synovial fluid via arthrocentesis includes the following potential benefits, EXCEPT: a. Provides relief in tense synovial effusions. b. Improve the delivery of nutrients to cartilage and surrounding tissues. c. Concomitant injection of corticosteroids can increase recurrence of joint effusion. d. Decrease joint intra-articular pressure preventing synovial capillary perfusion e. Remove white blood cells, a source of destructive enzymes, from the joint.

c. Concomitant injection of corticosteroids can increase recurrence of joint effusion.

A therapist immerses a patient's edematous ankle in cold water. When a body part comes in direct contact with the cold agent, the energy is transferred through a. Convection b. Evaporation c. Conduction d. radiation

c. Conduction

Which of the following bladder functions can be voluntarily controlled? a. Relaxation of internal sphincter muscle b. Contraction of the internal sphincter muscle c. Contraction of the external sphincter muscle d. Adenosine Hydrolysis e. Adenosine Triphosphate

c. Contraction of the external sphincter muscle

The following statements describe total contact casting, EXCEPT: a. Promotes ulcer healing by increasing the weight bearing surface area and allow patient to remain ambulatory. b. Are snug-fitting, below knee casts that protect insensitive limbs from repetitive trauma c. Contraindicated for superficial plantar ulcers in the presence of decreased or absent sensation. d. Is an effective therapy for healing chronic neuropathic plantar ulcers in individuals with chronic sensory neuropathies e. Contraindicated in deep foot ulcers where abscesses, osteomyelitis or similar deep infection or gangrene is present.

c. Contraindicated for superficial plantar ulcers in the presence of decreased or absent sensation.

The joints of the pectoral girdle include of the following, EXCEPT: a. Acromioclavicular b. Coracoarcromial c. Coracoclavicular d. Sternoclavicular

c. Coracoclavicular

Babinski's sign is a strong indication of a disorder of: a. Basal nuclei b. Reticulospinal tract c. Corticospinal tract d. Vestibulospinal tract e. Tectospinal tract

c. Corticospinal tract

The following statements describe 02 carrying capacity of the blood, EXCEPT: a. A shift of the oxyhemoglobin dissociation curve to the right facilitates the unloading of 02 from hemoglobin b. The 02 content of the blood is determined by the amount of hemoglobin in the blood and by the partial pressure of oxygen in the blood. c. Decreases concentration of carboxyhemoglobin produces a leftward shift of the curve impairing 02 delivery d. Acidosis and increased body temperature facilitate the unloading of 02 from hemoglobin and the diffusion of 02 from capillaries to muscle cells e. A shift of the oxyhemoglobin dissociation curve to the left impairs the amount of 02 extracted by muscle

c. Decreases concentration of carboxyhemoglobin produces a leftward shift of the curve impairing 02 delivery

Segmental development is evidence of the human torso's nerve supply of an orderly series of spinal nerves; i. e one pair of spinal nerve supplies this strip-like area of skin, and is known as a a. Myotome b. Terminal branches c. Dermatome d. Spinal nerves e. Vertebral nerves

c. Dermatome

In evaluating the client, the caregiver a. Initiate planning for extended treatment as necessary. b. Prepare and document the treatment plan and program c. Determines the client's current conditions his functional abilities and limitations d. Documents client's condition and functional outcome abilities e. Determines and selects the appropriate treatment techniques and equipment

c. Determines the client's current conditions his functional abilities and limitations

A patient in rehabilitation hospital begins to verbalize about the uselessness of life and the possibility of committing suicide. The most important therapist action is Ask nursing to check the patient every 15 minutes b. Discuss the situation's plan to his family c. Discuss the situation with the patient's case manager d. Review the patient's past medical history for signs and symptoms of mental illness e. Suggest the patient be placed on a locked unit

c. Discuss the situation with the patient's case manager

Credit units may be earned when undergoing the following programs/activities, EXCEPT: a. Professional chair holder (10 cu per chair) b. On the job local (5 cu per training) and international training (10 cu per training) c. Distance inter and intra school quiz competition (1 cu) d. Inventions (10-3 cu per invention e. International study/observation tour (6 cu per hour)

c. Distance inter and intra school quiz competition (1 cu)

Decreases fluid volume in the vascular system by increasing fluid and electrolyte excretion: a. Calcium channel blockers b. Alpha-adrenergic blockers c. Diuretics d. Beta-adrenergic blockers e. Angiotensin-converting (ACE) enzyme inhibitors

c. Diuretics

Which of the following statements describe the liver? I. It is the largest organ in the body weighing about one-fiftieth of the total body weight. II. Itis a solid, radish brown, pliant organ situated mainly on the left side of the body. III. It produces lymphocytes and is the principal residence of reticulo-endothelial cells of the body IV. It is in contact with the under surface of the dome of the diaphragm, thus enjoying the protection of the lower ribs. V. It maintains the amount of glucose present in the body at a constant level. a. IV and V only b. I, II and IV c. I, IV and V d. II and V only e. I, II and III

c. I, IV and V

Motions in the foot take place on certain joints. Which of the following statements is false? a. Forefoot abduction and adduction take place primarily at the talonavicular and calcaneocuboid joints b. Forefoot abduction and adduction take place primarily at the midtarsal joint c. Dorsiflexion and plantarflexion take place between the talus and the tibia within the ankle mortis d. Dorsiflexion and plantarflexion take place between the talus and the tibia and fibula within the ankle mortis e. Subtalar inversion and eversion take place primarily at the talocalcaneal, talonavicular, and calcaneocuboid joints

c. Dorsiflexion and plantarflexion take place between the talus and the tibia within the ankle mortis

The ability to move within a given posture without loss of balance: a. Harmony b. Steadiness c. Dynamic stability d. Symmetry e. order

c. Dynamic stability

A therapist orders a wheelchair for a patient with C4 quadriplegia. Which wheelchair would be most appropriate for the patient? a. Manual wheelchair with hand rim projections b. Manual wheelchair with lowered center of gravity c. Electrical wheelchair with sip-and-puff controls d. Manual wheelchair with friction surface hand rims e. Electrical wheelchair with joystick controls

c. Electrical wheelchair with sip-and-puff controls

Supplying the lower limb, this artery enters the thigh in the midline in front then descends vertically. a. Tibial artery b. Left femoral circumflex artery c. Femoral artery d. Sciatic artery e. Popliteal artery

c. Femoral artery

This artery enters the thigh in the midline in front, descends vertically and supplies the lower limb a. Left femoral circumflex artery b. Tibial artery c. Femoral artery d. Popliteal artery

c. Femoral artery

Immediate postoperative care following cardiac transplantation include the following EXCEPT: a. Typically, heart rate is maintained at about 80 beats per minute for the first 72 hours and 154. b. Oral diet is begun as soon as patient can tolerate it Most patients are weaned from mechanical ventilator assistance within 24 to 36 hours then tapered off c. First endomyocardial biopsy is performed 7 to 10 days after transplantation d. Endotracheal and orogastric tubes are removed within 24 to 36 hours

c. First endomyocardial biopsy is performed 7 to 10 days after transplantation

If there is anterior cruciate instability, the tibia will rock a. Backward b. Medially c. Forward d. Laterally e. Diagonally

c. Forward

The following characterize the Golgi Tendon Organs (GTO), EXCEPT: a. GTOs discharge nerve impulses that are transmitted over large, rapidly conducting afferent axons to the spinal cord and cerebellum b. GTO nerve impulses at the spinal cord excites inhibitory interneurons c. GTO nerve impulses at the spinal cord will not limit the force developed to tolerance by the tissues being stressed resulting in injury. s. GTO''s reside within muscle tendons near the points of attachment of the muscle fiber to the tendon e. Small bundle of muscle fibers produce tension that stimulate GTOs

c. GTO nerve impulses at the spinal cord will not limit the force developed to tolerance by the tissues being stressed resulting in injury.

The following procedure describe hip adduction joint measurement, EXCEPT: a. Movement arm remains parallel to anterior femur. b. Plane of motion frontal. c. Goniometer axis is centered over greater trochanter. d. Client should avoid trunk rotation. e. Client is positioned in supine or lying on side, knee extended

c. Goniometer axis is centered over greater trochanter.

Resolution No. 217, Series of 1992 delists the names of delinquent professionals from the rolls of registered professionals if the professional a. Had been delinquent in the payment of the annual registration fees for at least three (3) years from the year it was last paid. b. Had been delinquent in the payment of the annual registration fees for three (3) continuous years from the year it was last paid. c. Had been delinquent in the payment of the annual registration fees for five (5) continuous years from the year it was last paid. d. Had been delinquent in the payment of the annual registration fees for at least five (5) years from year it was last paid. e. Completes only 20 CPE units per annum

c. Had been delinquent in the payment of the annual registration fees for five (5) continuous years from the year it was last paid.

The following statements apply to elbow flexion range of motion, EXCEPT: a. Normal variation is between 120 to 160 degrees b. Subjects with little soft tissue have a hard end-feel c. Has hard end-feel on contact of muscle between arm and forearm d. Contact on the muscle between the forearm and arm can stop the motion e. Averages 145 degrees

c. Has hard end-feel on contact of muscle between arm and forearm

Dark bands/band's a. Are also called I bands b. M line is due to thick bulge in the thin filament c. Have an H band in the center d. Are preceded by a G band off the center e. Have a Z line in the middle

c. Have an H band in the center

Physiological fatigue is a central concern of ergonomics. To prevent undesirable static overload, the following improvements are recommended, EXCEPT: a. Improvements in equipment heights b. Layout changes c. Holding a single comfortable position d. Keep the body in a neutral posture and the muscle relaxed e. Braces, straps and supports to hold the tool to the hand

c. Holding a single comfortable position

Acute postoperative rehabilitation of lung transplant cases include the following, EXCEPT: a. ROM exercises progressing to transfer out of bed b. Adequate pain control, optimal positioning, and modification in coughing techniques c. Huff coughing with closure of the glottis d. Postural drainage with shaking or vibration provided the patient is stable e. Body positioning and mobilization

c. Huff coughing with closure of the glottis

Increase in resting membrane potential, making it move negative. a. Depolarization b. Velocity excitation c. Hyperpolarization d. Action potential propagation e. Variable conductance

c. Hyperpolarization

A neurotransmitter that increases K+ iron conductance will a. Depolarize the cell membrane b. Stimulate the neuron and propagate the action potential c. Hyperpolarize the membrane d. All of these e. Bring the potential near its firing level

c. Hyperpolarize the membrane

Changes in the capability to produce a given response during practice when augmented feedback about the performance is provided a. Initiative b. Response c. Learning d. Performance e. Function

c. Learning

Which of the following statements describe erythrocytes? I. These are formed in bone marrow where they pain through several stages before reaching maturity. II. A mature red cell is completely filled with hemoglobin, a highly specialized compound of protein and iron. III. The wall of each erythrocyte is made up of a number of ultimate compounds of carbohydrates and fats. IV. The life of an erythrocyte is about four months, and four million new cells every second of every minute must be produced. V. They are produced in red bone marrow and in the spleen and other lymphoid tissues. a. I, II and III b. II, III and IV c. II and V only d. II and IV only e. II, IV and V

c. II and V only

The following statements are true of the cruciate ligaments of the knee, EXCEPT: a. External rotation of the leg reduces forward movement of the tibia on the femur even if the anterior cruciate ligament is torn b. If the amount of forward movement of the tibia on the femur in internal rotation is equal to than in the neutral position, both the anterior cruciate ligament and the posterolateral portion of the joint capsule may be torn c. If forward movement with the leg in the neutral position both posterior cruciate ligaments and the posteromedial portion of the joint capsule may be damaged. d. If the tibia is pushed posteriorly and it moves backward on the femur, posterior draw sign in positive e. The anterior cruciate may be torn if anterior draw sign positive.

c. If forward movement with the leg in the neutral position both posterior cruciate ligaments and the posteromedial portion of the joint capsule may be damaged.

The following is true in a malunion fracture complication, EXCEPT: a. There was inadequate fracture reduction b. The fracture has healed incorrectly, leaving the bone deformed, weak, and possibly causing pain c. If malunion is detected before healing is complete, surgical correction is necessary to align the bone fragments d. If malunion is detected after healing is complete, surgical correction is necessary to obtain a more normal position for healing e. If the malunion is slight, it may not have to be corrected because the correction procedure could cause delayed union or nonunion

c. If malunion is detected before healing is complete, surgical correction is necessary to align the bone fragments

If there is tibial torsion in tibial torsion test, to what direction will the malleolar line face in relation to the parallel line? a. If there is internal tibial torsion, the malleolar line may face directly anteriorly, close to the parallel line b. If there is internal tibial torsion, the malleolar line may face directly anteriorly, close to the parallel line c. If there is internal tibial torsion, the malleolar line may face directly posteriorly, close to the parallel line d. If there is external tibial torsion, the malleolar line may face directly anteriorly, close to the parallel line e. If there is external tibial torsion, the malleolar line may face directly posteriorly, close to the parallel line

c. If there is internal tibial torsion, the malleolar line may face directly posteriorly, close to the parallel line

This muscle contracts maximally when flexing the hip beyond 90 degrees: a. Pectineus b. Rectus Femoris c. Iliopsoas d. Sartorius

c. Iliopsoas

The following statements describes Down Syndrome, EXCEPT: a. Deficit in eye-hand coordination laterality, and visual control were present in older children. b. The present of monosynaptic reflex during platform perturbations suggested that balance problem do not result from hypotonia but from defects within higher-level postural control mechanisms c. In 7-11 years of age group, those with Down Syndrome scored equally in running speed, balance strength and visual motor controls than the group without d. In children 1-6 years of age postures responses to loss of balance were slow and therefore insufficient for maintaining stability e. Attainment of early motor milestones is though to be delayed because of problems with ligamentous laxity in some joints, decreased strength, and hypotonia.

c. In 7-11 years of age group, those with Down Syndrome scored equally in running speed, balance strength and visual motor controls than the group without

The following are true of palmar prehension pattern, EXCEPT: a. Large objects may be held by widening the grip. b. This grasp is used to pick up small objects. c. In a glass jar, the entire palmar surface of the hand grasps around it, and the thumb closes in over the glass. d. The thumb opposes one or more of the other digits.

c. In a glass jar, the entire palmar surface of the hand grasps around it, and the thumb closes in over the glass.

If exercise tightness of the hip ligaments, muscles, or fasciae prevents complete extension of the hip joints, an individual commonly compensates for this limitation by a. Lengthening the vastus group b. Accentuating the anterior tilt of the pelvis and lumbar lordosis c. Increasing the posterior tilt of the pelvis d. Lumbar kyphosis

c. Increasing the posterior tilt of the pelvis

This tiny ophthalmic artery which accompanies the optic nerve and the anterior and middle cerebral arteries supply most of the cerebral hemisphere. a. Occipital Artery b. External Carotid Artery c. Internal Carotid Artery d. Subclavian Artery e. Optic Artery

c. Internal Carotid Artery

The following statements are true of the Tibialis Posterior, EXCEPT: a. Innervated by the tibial nerve b. The most deeply situated muscle of the calf c. Inverts and supinate of the subtalar joint only in dorsiflexion d. Inverts and assists in ankle plantar flexion

c. Inverts and supinate of the subtalar joint only in dorsiflexion

The nutritional component/s necessary for proper erythrocyte formation and malnutrition is/are: a. All of these b. Riboflavin (Vit B2) c. Iron and folic acid d. Folic acid e. Iron

c. Iron and folic acid

Which of the following is NOT true of the Deltoid? a. The nerve supply is by C5 and C6 via the circumflex nerve b. It can carry the arm higher than the horizontal position c. It is a powerful abductor of the humerus but cannot initiate that movement d. The middle fibers concern themselves with abduction alone

c. It is a powerful abductor of the humerus but cannot initiate that movement

Feedback information about the outcome of movements: a. Performance of Results b. Result Oriented c. Knowledge of Results d. Knowledge of Function e. Performance Orientation

c. Knowledge of Results

A deep sulcus that outlines and sweeps backward above the temporal pole, and then continues on the superolateral surface, almost horizontally backwards, marking the line along which the hemisphere became folded. The speech and hearing area are both closely associated with it. a. Medial b. Central c. Lateral d. Collateral e. Calcarine

c. Lateral

Chiasmal lesions could result in the following visual field defects, EXCEPT: a. Lesion involving both the optic nerve and the chiasm produces ipsilateral blindness and a temporal field defect in the other eye b. Transection of the an optic nerve result in ipsilateral monocular blindness c. Lesion of the right optic tract results in right homonymous hemianopsia d. Chiasmal lesion produces bitemporal hemianopsia

c. Lesion of the right optic tract results in right homonymous hemianopsia

This nerve supplies the Serratus Anterior Muscle and arises from the C5, C6 and C7 roots of the brachial plexus: a. Subscapular Nerve b. Musculocutateneous Nerve c. Long Thoracic Nerve d. Lateral Pectoral Nerve

c. Long Thoracic Nerve

The following are determinants of gait, EXCEPT: a. Knee flexion in stance b. Trendelenburg c. Medial displacement of the pelvis d. Pelvic rotation e. knee, ankle and foot rotation

c. Medial displacement of the pelvis

The following joint receptors provide the subcortical nuclei and cortex with constant information about position and movement, EXCEPT: a. Golgi mazzoni corpuscles b. Type lll Golgi type endings c. Merkel's disks d. Ruffini corpuscles

c. Merkel's disks

Deep tendon reflexes are a. Polysynaptic nerve response b. Excitatory transmission c. Monosynaptic reflex response d. Excitatory presynaptic potential e. Bisynaptic reflex pressure

c. Monosynaptic reflex response

therapist is treating a CVA patient with apraxia. All of the following techniques are commonly used to treat apraxia, EXCEPT: a. Repetition of tasks may be necessary b. The therapist should speak slowly and directly to the patient c. Multiple step commands should be used for activities of daily living d. A new task should be broken down into smaller components

c. Multiple step commands should be used for activities of daily living

Afferent component of muscle stretch reflex that provide CNS information about the length and rate of change in length of striated muscles. a. I band b. Intrafusal fiber c. Muscle spindle d. Extrafusal fiber e.Z band

c. Muscle spindle

The stability of the shoulder joint principally depends on: a. Capsule b. Bony configuration c. Muscles d. Ligaments

c. Muscles

The following conditions are all associated with spasticity, EXCEPT: a. Reflex irradiation b. Abnormal muscular co-contraction during voluntary movement c. Normal patterns of muscular coordination d. Reduction at the threshold angle at which the stretch reflex is elicited e. Hypersensitivity to various sensory input in addition to muscle stretch

c. Normal patterns of muscular coordination

Children with spastic cerebral palsy demonstrate pathological gait patterns as in the following, EXCEPT: a. Fixed contracture of the triceps surae muscle group is evident b. May be accompanied by anterior pelvic tilt during the stance phase c. Often accompanied by excessive knee flexion or hyperextension d. Decreased walking speed and stride length e. Abnormal joint motions including dynamic equinus or excessive ankle plantar flexion during the stance phase of gait

c. Often accompanied by excessive knee flexion or hyperextension

The following statements are true in the technique known as phonophoresis, EXCEPT: a. Medications commonly used include hydrocortisone, dexamethasone, salicylate and lidocaine, and frequently, hydrocortisone is combined with a gel base to formulate a 10% hydrocortisone solution b. Combined use of a drug-impregnated coupling medium with ultrasound to drive whole molecules of topically applied medication percutaneously into inflamed tissues c. Often applied via continuous ultrasound with intensity between 1.0 and 2.0 W.cm, at a frequency of 5 MHz, and with treatment duration of 5 minutes. d. The physical mechanism by which phonophoresis acts is not known e. Acts is a fast, painless, noninvasive alternative to local injection

c. Often applied via continuous ultrasound with intensity between 1.0 and 2.0 W.cm, at a frequency of 5 MHz, and with treatment duration of 5 minutes.

8. The muscle length at which the maximum tension is attained. A. Ascending limb b. Plateau region c. Optimal length d. Sarcomere length-tension relationship e. Descending limb

c. Optimal length

Postoperative rehabilitation of acute-posterolateral instability of the knee consist of the following, EXCEPT: a. After the cast is removed, a knee-ankle-foot-orthosis with a dial lock is applied b. Active extension is controlled because the arcuate complex has been advanced c. Orthosis is worn at all times and the dial lock is adjusted to allow extension of an additional 15 degrees each week d. The long-leg brace dial lock is adjusted to allow full flexion but to limit extension to about 60 degrees e. The subject begins partial weight bearing when the lock prevents only the final 15 degrees of extension

c. Orthosis is worn at all times and the dial lock is adjusted to allow extension of an additional 15 degrees each week

In the neuropathic extremity, a poor indicator of inflammation is: a. Edema or loss of function b. Heat and redness c. Pain and disturbance of function d. Redness and edema e. Edema or pain

c. Pain and disturbance of function

This lifting or stooping technique reduces the stress on both the back and the knees. The feet are positioned shoulder width part with one foot slightly in front of the other. One hand is placed on the forward thigh or a fixed object as the knees and hips are bent and then lowers him to pick up the item. The lift is completed by pushing off with the supporting hand as he stands up: a. Power lift b. Tripod lift c. Partial squat lift with support d. deep squat lift e. Diagonal lift

c. Partial squat lift with support

A powerful adductor and medial rotator of the arm, the clavicular head flexes the shoulder joint and from this position the sternal head extends the shoulder joint: a. Serratus Anterior b. Teres Major c. Pectoralis Major d. Pectoralis Minor e. Latissimus Dorsi

c. Pectoralis Major

Crystalloids and colloids solution in water a, Blood platelets b. White blood cells c. Plasma d. Macrophages e. Phagocytes

c. Plasma

The Problem Oriented Medical Record is a system based on the following, EXCEPT: a. Identification of a specific treatment plan b. Assessment of the effectiveness of the treatment plans c. Preparation of the informed consent d. Formation of present and past information about the patient e. Development of a specific, current problem

c. Preparation of the informed consent

The following statements describe congestive heart failure, EXCEPT: a. Some type of injury to the myocardium of ten initiates a progressive decline in myocardial function b. Characterized by decrease in cardiac pumping ability that leads to inadequate tissue perfusion and accumulation of fluid in the lungs and other organs c. Prognosis for patients with heart failure is often poor, with 5 year mortality rates d. Compensatory neurohumoral mechanism changes may actually exacerbate myocardial dysfunction by increasing cardiac work load e. Decreased sympathetic nervous system activation and increased activation of the rennin angiotensin system occurs as the heart fails

c. Prognosis for patients with heart failure is often poor, with 5 year mortality rates

Patients with anemia fatigue easily and treatments are modified accordingly. PT management include the following, EXCEPT: a. If exercise is indicated, pacing and training that distribute the intensity of the workload can be used to promote physiological recovery b. Mobilization in conjunction with gravitational stimulus to elicit hemodynamic responses to gravity c. Progressive training through low intensity but not by high intensity exercise d. Progressive training by alternating high-intensity exercise and low intensity exercise or rest e. Exercise stimuli such as walking and transferring to optimize ventilation, perfusion and to promote mucociliary transport.

c. Progressive training through low intensity but not by high intensity exercise

Receives and transmits impulses from tissues directly concerned with musculoskeletal movements and postures. a. Visceroceptors b. Musculoceptors c. Proprioceptors d. Exteroceptors e. Miessner's corpuscles

c. Proprioceptors

Differentiation of these three vesicles at the rostral end of the neural tube initiates brain development. a. Prosencephalon, mesencephalon and metencephalon b. Telencephalon, diencephalon, and mesencephalon c. Prosencephalon, mesencephalon and rhombencephalon d. Mesencephalon, rhombencephalon and metencephalon e. Prosencephalon, rhombencephalon and myelencephalon

c. Prosencephalon, mesencephalon and rhombencephalon

Dividing the body into right and left, flexion and extension occurs in this plane. a. Transagittal b. Horizontal c. Sagittal d. Pelvic rotation e. knee, ankle and foot rotation

c. Sagittal

Following statements characterize amyotrophic lateral sclerosis, EXCEPT: a. Characterized by degeneration of neurons in the motor nuclei of the cranial nerves and anterior gray horns of the spinal cord. b. A combined upper and lower motor neuron lesion that may involve the spinal or bulbar levels or both. c. Sensory disturbance is an integral part of the disorder. d. Involvement of the nuclei of the lower cranial nerves results in speaking and swallowing difficulty. e. Progressively fatal disease of unknown origin.

c. Sensory disturbance is an integral part of the disorder.

This cranial nerve supplies the muscles of expression, including the Buccinator: a. Fifth b. Eight c. Seventh d. Third e. Ninth

c. Seventh

Down syndrome exhibit various types of motor impairment that are as follows: EXCEPT: a. Utilize different strategies to control their movements. b. Movements are clumsy, i. e. slow and low efficacy. c. Shorter reaction times. d. Displays atypical sequences of motor development. e. Delays in motor performance at a very young age.

c. Shorter reaction times.

This type of traction immobilizes a body part for prolonged periods by attaching weighted equipment directly to the patient's bones, accomplished with pins, screws, wires or tongs: a. Weight Traction b. Skin Traction c. Skeletal Traction d. Manual Traction e. Equipment Traction

c. Skeletal Traction

Bilateral vestibular deficits results in the following, EXCEPT: a. None of these b. Oscillopsia c. Skew deviation d. Disequilibrium e. Postural instability

c. Skew deviation

A rehabilitation hospital assembles a team of health professions to after comprehensive care for patients following amputation. Which member of the team would be responsible for assisting the patient and family with financial matters and acting as a liaison with third party payers? a. Physical therapist b. Occupational therapist c. Social worker d. physiatrist e. vocational counselor

c. Social worker

This process moves Na+ out of the cell and K+ into the cell and requires an energy source of adenosine triphosphate: a. Passive distribution b. Chemical transduction c. Sodium-potassium pump d. Active distribution

c. Sodium-potassium pump

Characterized with hypertonicity associated with involuntary quick muscle contraction, increased muscle tone and muscle stretch reflexes: a. Chorea b. Tremors c. Spasticity d. Athetosis e. Dystonia

c. Spasticity

The two mode theory of vision asserts that: a. The degradation of perceptual and sensory systems are linked to potential postural problems b. The visual system acts not only via the clarity with which it "sees" but also via the information that is generated by the individual's motion through the environment c. Spatially distributed information comes via the ambient mode, responsible for orientation and locomotion, and the focal mode, responsible for object recognition and identification d. The somatosensory and visual systems are primarily sensitive to low frequency stimulation e. Visual perception is anchored to the optic array and emphasizes the important of optical information retrieved from various sectors of the optic array

c. Spatially distributed information comes via the ambient mode, responsible for orientation and locomotion, and the focal mode, responsible for object recognition and identification

Splints that hold the joint at some constant position: a. Dynamic-progressive b. Dynamic c. Static d. Static-Progressive e. Static-progressive

c. Static

If the tendon surface become irritated and rough, and if the sheath becomes inflamed and continues to press on the tendon, the following condition may be diagnosed: a. Tendinitis b. Epicondylitis c. Stenosis Tenosynovitis d. Tenobursitis e. Cystitis

c. Stenosis Tenosynovitis

The ability to recognize numbers or letters traced on the body: a. Sensory extinction b. Cutaneous tactile localization c. Stereognosis d. Graphesthesia e. Sensory inattention

c. Stereognosis

The posterior triangle of the neck is bounder by the following structures: a. The Scalene, Levator Scapulae, and Splenius b. Trapezius, Sternohyoid and Clavicle c. Sternocleidomastoid, anterior border of Trapezius, and the clavicle e. Splenius, Capitis, Sternohyoid, and clavicle e. Omohyoid, Sternocleidomastoid and Levator Scapulae

c. Sternocleidomastoid, anterior border of Trapezius, and the clavicle

The most effective stimulus for the muscle spindle to elicit the knee jerk is: a. Touch b. Needle prick c. Stretch d. Pain e. Stimulation of the proprioceptors

c. Stretch

The site of most frequent fracture of the upper end of the humeral shaft: a. Greater tubercle b. Bicipital groove c. Surgical neck d. Anatomical neck

c. Surgical neck

When defining the motion between the foot and leg, a common center of rotation is assumed with this joint primarily responsible for dorsiflexion/plantar flexion components: a. Metatarsophalangeal b. Talonavicular c. Talocrural d. Transverse tarsal e. Calcaneocuboid

c. Talocrural

Resolution No. 381, Series of 1995 standardized the guidelines and procedures for the implementation of the continuing professional education program, which includes the following, EXCEPT: The CPEC is composed of a chairman, from the members of the PRB, and two members, the first is the president of the PPTA and OTAP, the second is the president of the Association of PT and OT Schools b. The Continuing Professional Education (CPE) Council for PT and OT assists its corresponding Professional Regulatory Board (PRB) for PT and OT in implementations CPE program. c. The CPEC does not accept, approve applicants for accreditation of CPE providers d. The term of office of the chairman of each CPE council is co-terminus with his/her incumbency in the PRB. e. The term of office of the first and second member is co-terminus with his/her incumbency as officer of their respective organizations.

c. The CPEC does not accept, approve applicants for accreditation of CPE providers

The following statements describe the foot's lateral aspect, EXCEPT: a. The Peroneus brevis inserts into the styloid process of the fifth metatarsal b. The lateral malleolus is more posterior than the medial malleolus c. The anterolateral portion of the talar dome become palpable as the lateral malleolus rotates out from under the ankle mortis when the foot is dorsiflexed d. Directly behind the flare of the styloid process of the fifth metatarsal and in front of the cuboid lies a depression created by the peroneus longus as it runs to the medial plantar foot surface e. The peroneal tubercle lies on the calcaneus, distal to the lateral malleolus, a significant landmark because it separates the peroneus brevis and longus tendons

c. The anterolateral portion of the talar dome become palpable as the lateral malleolus rotates out from under the ankle mortis when the foot is dorsiflexed

Following statements are true of the corticospinal or pyramidal tract, EXCEPT: a. About 1/3 of the axons in the pyramidal tract arise from the primary motor cortex in areas 4 to 6. b. About 90 percent of the tracts decussate to the opposite side at the lower levels of the medulla. c. The fibers arise from the neurons in the ventral part of area 4 on the lateral surface of the hemisphere and from area 8. d. Concerned with skilled movements of the distal muscles of the limb

c. The fibers arise from the neurons in the ventral part of area 4 on the lateral surface of the hemisphere and from area 8.

The following statement describe the radius, EXCEPT: a. The radial shaft's crossing over the ulna is further aided by the shaft of the radius that is bowed a little lateral wards b. The head of the radius articulates with the capitulum of the humerus c. The important functions of the radius lie at the elbow d. The radial shaft crosses in front of the ulna while the radial head revolves to perform the movement of pronation e. The radius lies on a plane anterior to the ulna

c. The important functions of the radius lie at the elbow

With respect to tying a traction knot, the following statements are true, EXCEPT: a. To prevent fraying, tape the cord around the area you'll be cutting b. For safety, all traction knots are checked every eight hours c. The most commonly used traction knot is the slipknot d. The free end cord above the knot is secured by wrapping it with adhesive tape e. The knot must also wrapped and covered with adhesive tape

c. The most commonly used traction knot is the slipknot

The following statements describe repair of acute instability, EXCEPT a. Advancement of the posterior oblique ligament is a reconstructive procedure that is almost always used b. Peripheral tears of the medial meniscus is salvaged buy suturing the structure to the capsular ligaments c. The need to drill holes in bone to reattach ligaments is seldom necessary inasmuch as the periosteum is usually intact and will hold sutures well d. Repair of an acute case by direct suture is usually avoided e. Ligament stapling is avoided because it tends to "strangle"

c. The need to drill holes in bone to reattach ligaments is seldom necessary inasmuch as the periosteum is usually intact and will hold sutures well

Intervertebral joint palpation provides the clinician valuable information about joint behavior to apply a force to a joint and to evaluate the joint's response to that force. In the process, he/she must appreciate the following, EXCEPT: a. The amount of movements produced at the joint and presence activity evoked during the movement b. The speed and direction at which it is applied c. The paint produced by the movement d. The way in which the joint moves or resists movement in response to the given force e. The amount of force applied to the muscle inserting into the joint

c. The paint produced by the movement

In Allen Test, if the hand does not react, of it if flushes very slowly a. The released artery is not occluded b. The released vein is not occluded c. The released artery is partially or completely occluded d. The released vein is partially or completely occluded e. The released artery and vein are partially or completely occluded

c. The released artery is partially or completely occluded

The following apply to Distraction, EXCEPT: a. Relieves pain in the cervical spine by decreasing pressure on the joint capsule around the facet joint b. May help alleviate muscle spasm by relaxing the contracted muscle c. The test faithfully reproduce pain referred to the upper extremity from the cervical spine d. The test demonstrates the effect that neck traction might have in relieving pain e. Relieves pain due to a narrowing of the neural foramen by widening the foramen

c. The test faithfully reproduce pain referred to the upper extremity from the cervical spine

Stress fractures of the femoral neck are generally characterized by the following, EXCEPT: a. Radiographs are often negative b. Injection with a local anesthetic will provide the complete relief of pain c. There is sudden onset of pain and a recent change in activity level d. In the early stages, localized pain that worsens with activity and improves with rest is present e. Localized tenderness may occur at the greater trochanter

c. There is sudden onset of pain and a recent change in activity level

An upper limb tension test is considered positive if the following are present, EXCEPT: a. The patient's responses are different from what is expected for asymptomatic subjects b. It reproduces the patient's symptoms c. There is tissue resistance or a decrease in the ROM on one side of the body as compared with the other d. The test responses can be altered by a sensitizing maneuver that provokes an increase in symptoms e. The symptoms are aggravated by movements that are generally towards from the site of restriction or symptoms

c. There is tissue resistance or a decrease in the ROM on one side of the body as compared with the other

A stretching method in which the muscle is slowly elongated to tolerance, short of pain, nd the position held with the muscle in this greatest tolerated length. a. Increase in gait speed is achieved by lengthening strides rather than by increasing the number of steps per minute b. Loss of mobility at the ankle and foot, lack of distal muscular control, and absent proprioception all contribute to altered gait characteristics. c. These individuals have the potential for very high function because of their retention of normal hip and knee control d. There is substantial reduction in free walking speed, as a result of both diminished stride length and cadence e. Strength deficits in the remaining musculature often limit ambulation ability

c. These individuals have the potential for very high function because of their retention of normal hip and knee control

The most dilute luminal fluid in the presence of anti-diuretic hormone is found in the a. Proximal tubule b. Proximal convoluted tubule c. Thick ascending limb of the loop of Henle d. Distal convoluted tubule e. Thin descending limb of the loop of Henle

c. Thick ascending limb of the loop of Henle

Foot dorsiflexor are composed of: a. None of these b. Tibialis Anterior, Extensor Digitorum Brevis and Extensor Hallucis Brevis c. Tibialis Anterior, Extensor Digitorum Longus and Extensor Hallucis Longus d. Peroneous Tertius, Peroneus Brevis and Extensor Digitorum Brevis

c. Tibialis Anterior, Extensor Digitorum Longus and Extensor Hallucis Longus

In carpal tunnel syndrome, alteration in the size of the structures under the transverse carpal tunnel ligament such as occurs with inflammation, edema or fascial scarring can affect the perineural vasculature. This inflammation process can result in the following conditions, EXCEPT: a. Fibroblastic proliferation secondary to chronic edema my result in interneural fibrosis of the median nerve b. Self perpetuating cycle of hypoxia c. Tingling in one or more digits in the median nerve distribution when the patients actively maintain maximal wrist flexion for one minute d. Leakage of edema from damaged capillary endothelium.

c. Tingling in one or more digits in the median nerve distribution when the patients actively maintain maximal wrist flexion for one minute

The following statements describe the prosthesis of an above knee amputee, EXCEPT: a. When a SACH foot is used, it is set at about 90 degrees with respect to the shank b. At faster speeds, a knee damper assists in improving control of the calf momentum c. To improve foot clearance, the unaffected pelvis may be elevated d. Due to lack of propulsion on the calf, the pelvis drops in late stance on the prosthetic side e. The prosthetic limb's forward motion must be generated at a site other than the calf

c. To improve foot clearance, the unaffected pelvis may be elevated

The following statements are true of the Carpal Tunnel, EXCEPT: a. Carpal bones border the tunnel posteriorly b. The transverse carpal ligament runs between four prominences and form a fibrous sheath containing it anteriorly within a fibro-osseous tunnel c. Transports the ulnar nerve and finger flexor tendons from the forearm to the hand d. Proximally defined by the pisiform and the tubercle of the navicular e. Distally defined by the hook of the hamate and the tubercle of the trapezium

c. Transports the ulnar nerve and finger flexor tendons from the forearm to the hand

A therapist uses a bandage to construct a sling for a patient with a painful shoulder. The most appropriate bandage to utilize is a. Four-inch elastic bandage b. Six inch roller gauze Triangular bandage c. Two inch roller gauze d. Six-inch elastic bandage

c. Two inch roller gauze

A 75 year-old patient with bilateral transfemoral amputations works on ambulation activities in an-outpatient rehab clinic. Which type of assistive device would be most appropriate to utilize during the training session? a. Cane b. Two forearm crutches c. Walker d. two canes e. quad cane

c. Walker

Examples of static load include the following, EXCEPT: a. Standing upright for long periods of time b. Holding the arms unsupported out in front to do assembly work c. Walking for long periods of time d. Continuously holding a tool in one hand and the product in the other e. Reaching overhead and looking upward

c. Walking for long periods of time

The muscle/s which keep the humeral head apposed to the glenoid cavity when carrying weight: a. Supraspinatus b. Infraspinatus c. all of these d. teres minor

c. all of these

Unusually violent and flinging motions of the limbs: a. Chorea b. Dystonia c. ballisimus d. spasticity e. athetosis

c. ballisimus

a. Popliteal b. Dorsalis pedis c. femoral d. tibial e. common peroneal

c. femoral

When using functional electrical stimulation to minimize fatigue during repetitive muscle activation, we want the following setting to produce targeted forces. a. Highest frequency and highest intensity b. Lowest frequency and highest intensity c. lowest frequency and lowest intensity d. highest frequency and lowest intensity

c. lowest frequency and lowest intensity

A therapist prepares to conduct a manual muscle test of the hip flexors. Assuming a grade of poor, the most appropriate testing position is a. Sitting b. Supine c. sidelying d. prone e. standing

c. sidelying

During ambulation, the leg is brought into the trailing position by the following, EXCEPT: a. Extension of the hip b. Anterior tilting of the pelvis c. tightening of the rectus abdominis d. extension of the lumbar spine

c. tightening of the rectus abdominis

The proximal carpal row consists of the following, EXCEPT: a. Lunate b. Scaphoid c. trapezium d. pisiform e. triquetrum

c. trapezium

Cerebrospinal fluid is formed primarily by specialized tissue in the ventricles called a. Pia meter b. Meninges c. Anterior Commissural d. Choroid plexus e. Dura mater

d. Choroid plexus

In MMT, when there is visible palpable or muscle contraction but no joint motion is produced, the muscle is graded as: a. 3 b. 4 c. 0 d. 1 e. 2

d. 1

The hormone regulating Ca++ metabolism that is produced by the kidney is a. Estrogen b. Thyroid c. Parathyroid d. 1-25 OH2 cholacalciferol e. Calcitonin

d. 1-25 OH2 cholacalciferol

The amount of calories required concentrating 1 osmole of substance tenfold? a. None of these b. 1200 calories c. 1600 calories d. 1400 calories

d. 1400 calories

For a stroke volume of 70 ml and a heart rate of 70 beats per minute, the cardiac output is equal to a. 2,450 ml b. 140 ml c. 1,633 ml d. 4,700 ml e. 1,225 ml

d. 4,700 ml

Licenses of registered physical and occupational therapists shall be renewed upon presentation of credit units earned in the past three years totaling a. 30 credit units b. 50 credit units c. 70 credit units d. 60 credit units e. 80 credit units

d. 60 credit units

Signs of cardiac dysfunction include the following, EXCEPT: a. A heart rate that is either excessively high of exceptionally low during exercise b. An increase in diastolic blood pressure during exercise that is greater than 15 to 20 mm Hg c. Electrocardiographic changes such as dysrythmias or ST-segment depression d. A diastolic blood pressure that does not rise progressively as work level increases e. A systolic blood pressure that falls during exercise

d. A diastolic blood pressure that does not rise progressively as work level increases

The following statements characterize, fibromyalgia syndrome, EXCEPT: a. Frequently misdiagnosed and confused with myofascial pain syndrome, polymyositis, juvenile rheumatoid arthritis or systemic lupus erythematosus. b. A chronic pain disorder of unknown etiology. c. There are widespread musculoskeletal aches and pains, stiffness and general fatigue. d. A non-rheumatic disorder. e. Median age of onset is from 29 to 37 years and medical presentation is 34 to 53 years.

d. A non-rheumatic disorder.

These muscles make up the thenar eminence. a. Flexor Pollicis Brevis, Flexor Pollicis Longus and Opponens Pollicis b. Abductor Pollicis Brevis, Flexor Pollicis Longus and Abductor Pollicis Longus c. Abductor Pollicis Brevis, Abductor Pollicis Longus, and Opponens Pollicis d. Abductor Pollicis Brevis, Opponens Pollicis and Flexor Pollicis Brevis e. Flexor Pollicis Longus, Opponens Pollicis and Abductor Pollicis Longus

d. Abductor Pollicis Brevis, Opponens Pollicis and Flexor Pollicis Brevis

Functions of disc include: a. Permit two types of movement to occur simultaneously b. Lessen shock in a joint and adjust bone articulating surface of different shape to one another c. Assist in lubrication of articular surfaces d. All of these

d. All of these

External fixation is used to stabilize fractures that are difficult or impossible to immobilize with casts, traction pins, or plates. It is indicated in the following conditions, EXCEPT: a. An acutely infected open fracture b. A massive open fracture, with extensive soft tissue injury c. Intra-articular fracture d. Arthrodesis e. A comminuted fracture

d. Arthrodesis

As proposed by Cyriax, the scheme of selective tension testing is a clinical system of diagnosis of painful problems of soft issues and is conducted in the following manner, EXCEPT: a. Resisted contractions b. Palpation c. Active motion d. Assisted contractions e. Passive motion

d. Assisted contractions

his stretching technique uses bouncing or quick, jerking movements imposed on the muscles to be stretched and can theoretically exceed the extensibility limits of the muscle in an uncontrolled manner and cause injury: a. Compound stretch b. Proprioceptive neuromuscular facilitation c. Sustained stretch d. Ballistic stretch e. Static stretch

d. Ballistic stretch

The following statements describe the wrist: EXCEPT: a. In abduction, the scaphoid, lunate, and triquetrium shift medially. b. The scaphoid, lunate, and triquetrium shift laterally in adduction. c. Due to reinforcement provided by fibers, the corpus is forced to move with the radius as a unit during pronation-supination. d. Because the joint is ellipsoidal, movements can occur only in the three axes of the ellipse. e. It can be flexed, extended, abducted or adducted.

d. Because the joint is ellipsoidal, movements can occur only in the three axes of the ellipse

Stimulation of parasympathetic nervous system will result in a. Tachycardia b. Increased blood pressure c. Cerebrovascular accident d. Bradycardia e. Increased cardiac contractility

d. Bradycardia

The adult brain consists of the a. Pons, Medulla Oblongata and Medulla b. Cerebrum, Basal Ganglia, Thalamus and Epithalamus c. Cerebellum, Ventricles and Medulla Oblongata d. Cerebrum, Cerebellum and the Brain stem e. Hypothalamus, Cerebellum, Brain Stem, Mesencephalon and Medulla Oblongata

d. Cerebrum, Cerebellum and the Brain stem

Inhibitory post-synaptic potentials (IPSP) are produced with hyperpolarization of the cell membrane. This is due to: a. Sulfate influx b. Potassium influx c. Magnesium influx d. Chloride ion influx e. Sodium influx

d. Chloride ion influx

A combination of the four primary movements in which each succeed one another. a. Lateral rotation in the horizontal or transverse plane b. Rotation in the sagittal plane c. Adduction in the coronal or frontal plane d. Circumduction e. Medial rotation in the coronal plane

d. Circumduction

Which of the following structures prevent a lateral or medial dislocation of the knee joint? a. Intercondylar eminences b. All of these c. Cruciate ligaments d. Collateral ligaments

d. Collateral ligaments

Theoretical relationships are established between the primary concept to be measured and one or more other concepts in this type of validity. a. Prescriptive b. Content c. Predictive d. Construct e. Face

d. Construct

A therapist observes a patient exercising. It becomes obvious that the patient is not challenged by the exercise routine and is receiving little benefit from the therapy. The most appropriate action for the therapist would be to a. Modify the patient's program as necessary b. Tell the patient they are no longer benefiting from physical therapy c. Remain silent since it is another therapist's responsibility to modify the treatment plan d. Consult with the supervising therapist and suggest treatment alternatives e. Discuss with the consulting doctor your observation

d. Consult with the supervising therapist and suggest treatment alternatives

Loss of the lower motor neuron to a muscle may result in the following, EXCEPT: a. Progressive atrophy b. Reflex response failure c. Flaccidity d. Contractile myofibrils may be replaced with fibrous connective tissue in about 6 months.

d. Contractile myofibrils may be replaced with

Frowning muscle: a. Procerus b. Nasalis c. Frontalis d. Corrugator supercilli

d. Corrugator supercilli

The following are part of the extrapyramidal system, EXCEPT: a. Cerebellospinal tract b. Vestibulospinal tract c. Reticulospinal tract d. Corticospinal tract

d. Corticospinal tract

Neurodevelopment treatment (NDT) is the most commonly used approach to treatment with cerebral palsy. The following statements describe NDT, EXCEPT: a. Treatment outcomes are supposed to be achieved through physical handling of the child during movement, giving the child more normal sensorimotor experiences b. As the child gains postural control, the therapist gradually withdraws support c. This approach focuses on encouraging and building upon normal movement patterns and normal postural reactions while trying to reduce abnormal movements d. Current interpretations of NDT does not include the importance of functional skill practice in treatment e. It is implicit in NDT that improved postural control will lead to improvement in functional skills without necessarily working on these specific skills

d. Current interpretations of NDT does not include the importance of functional skill practice in treatment

The exchange of ions across the membrane when a stimulus is applied. a. Repolarization b. Impulse conduction c. lonization d. Depolarization e. Irradiation

d. Depolarization

In comparison to motor stimulation, sensory nerve excitation, irrespective of waveform, require less of the following, except: a. Phase charge b. Peak voltage c. Total pulse charge d. Differential amplitude e. Peak current

d. Differential amplitude

Person-level problems characterized by the inability to perform any of the activities considered usual for a human being, such as limitations in walking or limited ability to communicate: a. Disorder b. Infirmity c. Impairments d. Disabilities e. Affliction

d. Disabilities

The motion that the scapulothoracic joint provides result in the following statements, EXCEPT: a. Allow the deltoid to function above 90 degrees of arm elevation b. Increase the range of arm motion c. Provides glenohumeral stability for work in overhead position d. Does not allow elevation of the body, i.e. in sitting push-ups

d. Does not allow elevation of the body, i.e. in sitting push-ups

Disturbance in the execution of speech and often occurs without a disorder of language: a. Agnosia b. Aphonia c. None of these d. Dysarthria

d. Dysarthria

A frame of reference for balance and head control that provides spatial coordinates for limb and body-segment positions: a. Exocentric b. Proprioceptive c. Gravitational d. Egocentric e. Geocentric

d. Egocentric

The following statements characterize syringomyelia, EXCEPT: a. There is no sensory impairment in the lower extremities b. There is progressive cavitation around or near the central canal of the spinal cord. c. There is loss of pain and temperature sense with a segmental distribution in both upper extremities. d. Even in later stages, paralysis and muscle atrophy of the segment involved does not occur. e. Touch and pressure sense in the affected parts are preserved.

d. Even in later stages, paralysis and muscle atrophy of the segment involved does not occur.

To test this muscle, the proximal IP joint of the hand is stabilized a. Interossei b. Flexor Digitorum Superficialis c. Lumbricals d. Flexor Digitorum Profundus e. Pronator Teres

d. Flexor Digitorum Profundus

This ligament maintains the forward concavity and forms a bridge deep to which pas the numerous tendons that flex the wrist and fingers: a. Annular ligament b. Denticulate ligament c. Intraarticular ligament d. Flexor retinaculum e. Extensor retinaculum

d. Flexor retinaculum

The shape of articulating surfaces of bi-axial joints are either a. Plane or Saddle b. Condyloid or ellipsoid c. Pivot or condyloid d. Hinge or pivot e. Ball and socket

d. Hinge or pivot

A functional map of the motor cortex resembling an image of the body turned upside down and reversed left to right. a. Brodmann's area b. Hemisphere c. Parietal Lobe d. Homunculus e. Premotor cortex

d. Homunculus

When measuring elbow flexion, the stationary arm of the goniometer should be aligned along the a. Radius b. Fifth metacarpal c. Fifth phalanx d. Humerus e. Ulna

d. Humerus

The Ulnar Nerve innervates the following structures, EXCEPT: a. All interossie b. Medial half of the Flexor Digitorum Profundus c. Adductor Pollicis d. Palmaris Longus e. All Hypothenar

d. Palmaris Longus

Which of the following are TRUE of the palmar aponeurosis? I. The Palmaris Longus inserts into this triangular fibrous sheet that occupies the palm of the hand. II. It protects underlying structures and prevents the palm of the hand from being readily pinched. lll. The aponeurosis is present only when the Palmaris Longus is present. IV. Allow the palm of the hand to be readily pinched. V. The aponeurosis is present even when the Palmaris Longus is absent. a. I, III and IV b. II and V only c. I, and II only d. I, II and IV e. II, III and IV

d. I, II and IV

Which of the following statement describe the heart? I. During development, the heart undergoes rotation so that its right side is carried backwards and its left side forwards. II. It is about the size of a clenched fist and occupies central position in the thoracic cavity. IIl. It lies behind the body of the sternum and in front of the middle four thoracic vertebra (T. 5,6,7 and 8) IV. The left ventricle forms the posterior surface and occupies most of the superior border. V. The right ventricle occupies most of the anterior surface and forms all but the extremities of the inferior border. a. II and V only b. I, II and IV c. I and IV only d. II, I and V e. II, III and IV

d. II, I and V

Cervical root compression syndromes are illustrated by the following, EXCEPT: a. There is localized paraspinal pain and tenderness which may precede extreme pain and which indicates a focus of disease proximal to the shoulder joint b. Usually involves one of the three lower cervical roots, C6, C7, or C8 c. Gentle manipulation of the neck reproduces pain felt below the elbow, and duplicates paresthesia in the hand d. If moderate pressure over the spine enhances pain felt the extremity impingement may be from other source e. Often, neck movements are restricted

d. If moderate pressure over the spine enhances pain felt the extremity impingement may be from other source

The following are true of Tetanic contractions, EXCEPT: a. Serial, continuous contractions, in which individual contractions cannot be distinguished b. Series of increasingly stronger twitch contractions occurring in response to repeated contractions c. They occur in response to a bombardment of the muscle by rapid, successive stimuli d. Incomplete Tetanic contractions may convert to a normal contraction if the excessive stimulation ceases abruptly

d. Incomplete Tetanic contractions may convert to a normal contraction if the excessive stimulation ceases abruptly

Principles of prevention of cumulative trauma injury for the hands and wrists include the following, EXCEPT: a. Eliminate direct pressure on the palm b. Design tools to keep wrist in the "neutral" position c. Isolate vibration d. Increase excessive or unnecessary grasping force where possible e. Reduce the number of repetitions

d. Increase excessive or unnecessary grasping force where possible

Following are changes in the mechanical properties of muscle fibers occurring in patients with spasticity, EXCEPT: a. Contraction times of hand muscles and gastrocnemius muscles are diminished b. Increased muscle tone in the antigravity muscles c. Structural changes such as the appearance of target fibers d. Increased levels of muscle fiber atrophy e. Predominance of type | fibers in the gastrocnemius muscle months after stroke, when spasticity of cerebral origin has been established

d. Increased levels of muscle fiber atrophy

The immobile but not rigid union of this ligament provides security to the ankle joint: a. Calcaneocuboid b. Superior tibiofibular c. Subtalar d. Inferior tibiofibular ligament e. Interosseous tibiofibular ligament

d. Inferior tibiofibular ligament

These muscles extend the arm and are the chief lateral rotators. a. Supraspinatus and Subscapularis b. Infraspinatus and Teres Major c. Subscapularis and Pectoralis Major d. Infraspinatus and Teres Minor e. Teres Major and Latissimus Dorsi

d. Infraspinatus and Teres Minor

The following characterizes straight lateral knee instability, EXCEPT: a. In isolated injury, the foot is fixed to the surfaces, with the knee in slight flexion b. Structures involved include the lateral collateral ligament and the middle third of the lateral capsule. c. A lateral force contacts the medial aspect of the knee d. Involved structures include the medial collateral ligament and middle third of the medial capsule e. Many of these lesions do not occur as isolated injuries.

d. Involved structures include the medial collateral ligament and middle third of the medial capsule

If two forces are pulling from the same point, the resultant force: a. Decreases as the angle between the two forces increase b. Is the sum of the two forces c. Reaches a maximum when the forces are on the same line and acting in opposite direction d. Is the diagonal of the parallelogram e. Increases as the angle between the two forces decreases

d. Is the diagonal of the parallelogram

The following statements describe the Triceps Brachii, EXCEPT: a. The long head may help also in extending the shoulder joint. b. The long and lateral heads lie side by side and together are comparable to the Biceps. c. Of the three heads, only the long head arises from the scapula. d. It acts in a lever system of the Second Class but its lever arm is extremely short. e. The medial head has fleshy origin from the back of the shaft similar to the Brachialis

d. It acts in a lever system of the Second Class but its lever arm is extremely short.

The following characterizes the patella, EXCEPT: a. Lateral dislocation is resisted by the lateral lip of the trough being projected forwards b. It maintains a constant relationship with the fibular as the knee is flexed c. Its posterior surface is articular since it lies in front of the expanded lower end of the femur. d. It is easily dislocated medially due to the powerful extensors of the knee that pull obliquely and chiefly from the lateral side of the thigh e. Enhances the power of knee extensors by increasing the leverage of that muscle.

d. It is easily dislocated medially due to the powerful extensors of the knee that pull obliquely and chiefly from the lateral side of the thigh

The following statements are true of biceps brachii, EXCEPT: a. A simultaneous flexion of the shoulder and elbow and supination of the forearm will result from an isolated, unopposed contraction of the biceps. b. When the forearm is supinated, it acts in flexion of the elbow with or without a load, in slow or in fast movements in either concentric or eccentric contraction c. Its effectiveness as a supinator is almost four times as effective as the supinator at an angle of 90 degrees d. It is most effective as a supinator when the elbow is flexed at about a 145 degree angle. e. Its effectiveness as a supinator diminishes as the elbow is extended

d. It is most effective as a supinator when the elbow is flexed at about a 145 degree angle.

Craniosacral therapy is a relatively new evaluation and treatment approach and assumes the presence of craniosacral motion to be as follows, EXCEPT: a. An inherent rhythmical motion in human and animals different from any other rhythm in the body b. Restricted craniosacral motion and presence of pathology in the body can guide corrective treatment to produce improvements in pain and dysfunction c. It is a physiological motion because it is unconscious and voluntary d. It is unstable and fluctuates in response to exercise, emotion and rest e. It is claimed to changed in response to trauma, autism or learning disability

d. Itis unstable and fluctuates in response to exercise, emotion and rest

The following statements are true of the Pituitary Gland, EXCEPT: a. Its rich blood supply is derived from the small branches of the Right and Left Internal Carotid Arteries b. It is made of the Anterior and Posterior Lobe that function differently c. It is a pea-size gland hanging by a stalk, the infundibulum, from the floor of the third ventricle of the brain, just behind the optic chiasm d. Its shape and appearance resembles a miniature pine cone, projecting backwards from the back of the roof of the ventricle.

d. Its shape and appearance resembles a miniature pine cone, projecting backwards from the back of the roof of the ventricle.

Ergonomic design in office settings has become increasingly important, particularly because of the increased use of computers. The following are acceptable recommendations, EXCEPT: a. Feet flat on the floor or footrest to provide stability b. Top of monitor placed at eye level to allow proper head and neck position c. Padded wrist rest to reduce arm and shoulder discomfort d. Keyboard placed at 15 degree from elbow height with a slight incline e. Hard copy holder close to monitor to reduce eye motions and discomfort and allow proper neck posture

d. Keyboard placed at 15 degree from elbow height with a slight incline

The following statements characterize the nervous system, EXCEPT: a. The speed at which a nerve impulse travels along the length of an axon is related to the diameter of the axon b. In a peripheral nerve, the average conduction velocity of large motor and sensory fibers is about the same c. Impulse conduction is faster in axons with myelin sheath d. Larger axons conduct impulses at a slower velocity

d. Larger axons conduct impulses at a slower velocity

A deep sulcus that outlines and sweeps backward above the temporal pole, and then continues, on the superolateral surface, almost horizontally backwards, marking the line along which the hemisphere became folded. The speech and hearing are both closely associated with it: a. Collateral Sulcus b. Central Sulcus c. Calcarine Sulcus d. Lateral Sulcus

d. Lateral Sulcus

The aorta leaves this chamber of the heart. a. Right ventricle b. It bypasses the heart c. Right auricle d. Left ventricle e. Left auricle

d. Left ventricle

A solute with a high portion coefficient is a. Moves through the cell membrane with difficulty b. Where saturation can occur c. Directed against a chemical gradient d. More soluble in fat than in water e. More soluble in water than in fat.

d. More soluble in fat than in water

Vestibular influence on the geocentric frame of reference are as follows, EXCEPT: a. Detection of head acceleration b. Tibialis anterior muscle activity increases during ambulation when head is tilted backward. c. Otolith organs provide reference for head position with respect to earth-vertical d. No influence on trunk and limb stability when head position changes e. Used for repositioning the eyes based on changes in head orientation

d. No influence on trunk and limb stability when head position changes

Abdominal muscle exercise during pregnancy and at least 8 weeks post birth should be chosen with care because of the following reasons, EXCEPT: a. There is a noted increase in rectus abdominis muscle separation width, length, and angles of insertion b. Decrements in abdominal muscle function can parallel in time the structural adaptations as pregnancy progressed c. The ability to stabilize the pelvis against resistance can be decreased as pregnancy progresses and remain compromised postbirth d. No reversal in rectus abdominis muscle separation may be found by 4 weeks postbirth e. Continued functional deficits may be found in parallel with incomplete resolution of structural adaptations post birth

d. No reversal in rectus abdominis muscle separation may be found by 4 weeks postbirth

This/These statement/s describes the excitability property of the neuron: a. It is most excitable when the membrane potential is reduced to its firing level b. It is excitable at all times c. It can be stimulated again at the spike of the action potential d. None of these e. All

d. None of these

When an injured site is immobilized in a cast, the following statements are true in the care of a casted lower extremity, EXCEPT: a. Flexing and extending the toes is an exercise to test nerve function b. To check for circulation, briefly press on the large toenail until it turns white. Let go, and observe if the normal pink color returns quickly c. Excess swelling may be prevented by keeping the casted leg elevated on pillows, above chest level d. Occasionally, when the skin under the cast feels itchy, an object may be inserted to try to relieve the itching. e. If the toes are cold, cover them. Notify the doctor if that doesn't warm them

d. Occasionally, when the skin under the cast feels itchy, an object may be inserted to try to relieve the itching.

The sensory cranial nerve supply of the eyeball is a. Fourth (Trochlear) b. Third (Oculomotor) c. Sixth (Abducens) d. Opthalmic division of the fifth (Trigeminal) e. Seventh (Facial)

d. Opthalmic division of the fifth (Trigeminal)

These mechanoreceptors are found in the skin and connective tissues surrounding bones and joints to detect vibration: a. Muscle spindle b. Ruffini's end organs c. Miessner's corpuscles d. Pacinian corpuscles e. Krause's end bulb

d. Pacinian corpuscles

The following statements describe Trochanteric bursitis, EXCEPT: a. Treatment usually consists of rest, immobilization, and local heat b. There is immediate relief of pain after peritrochanteric injection with corticosteroid and local anesthetic c. Results from friction between the bursa and the greater trochanter d. Pain and tenderness may develop in the medial thigh, groin, and the gluteal areas e. There is marked tenderness to deep palpation immediately above or posterior to the greater trochanter

d. Pain and tenderness may develop in the medial thigh, groin, and the gluteal areas

Terminal branches of the basilar artery: a. Vertebral arteries b. Posterior communicating arteries c. Superior cerebellar arteries d. Paired posterior cerebral arteries

d. Paired posterior cerebral arteries

Foot orthotic devices inserted between the foot and shoe to modify foot biomechanics are used in clinical settings for conditions aggravated by excessive pronation which include the following, EXCEPT: a. Posterior tibial tendinitis b. Anterior tibial tendinitis c. Achilles tendinitis d. Patellofemoral pain syndrome e. Plantar fasciitis

d. Patellofemoral pain syndrome

The following statements are true of the condylar type of total knee prosthesis, EXCEPT: a. Indicated for deteriorated, but not completely destroyed knee joint b. Indicated for patients that have no functioning ligaments in the knee c. Allows some adduction, abduction and rotation d. Patient must have normal knee ligaments e. Consists of metal rollers fixed onto the femoral condyles and high-density polyethylene cups located on the tibia

d. Patient must have normal knee ligaments

The exacerbating-remitting pattern of multiple sclerosis is characterized by: a. A steady worsening of symptoms over time b. Spasticity, fatigue and bladder dysfunction c. Periods of impairment followed by partial remission of symptoms d. Periods of impairment followed by full or partial remission of symptoms e. Evidence can be shown from the onset of their symptoms

d. Periods of impairment followed by full or partial remission of symptoms

Mobility aids such as a cane may be used other than in terms of biomechanical forces. The following demonstrate the timing relationships between applied cane forces and duration of stance, EXCEPT: a. Patients with Parkinson's are comfortable even though the force applied was less than that needed for physical support of the body b. Persons with above know amputations applied small cane force prior to the stance phase with the prosthesis, suggesting the cane provided sensory information before weight bearing. c. Patients with ankle arthropathy applied peak force late in the stance phase of the disable limb suggesting the cane was used to push forward. d. Persons with lowest extremity amputations shift their weight from a cane to prosthesis to avoid residual limb orientation at the limb/socket interface. e. Patients with degenerative joint disease of the hip applied an initial peak thrust early in the stance phase, suggesting that the cane was used for restraint.

d. Persons with lowest extremity amputations shift their weight from a cane to prosthesis to avoid residual limb orientation at the limb/socket interface.

All carpal bones have four articulating surfaces, EXCEPT: a. Triquetrum b. Scaphoid c. Hamate d. Pisiform e. Trapezium

d. Pisiform

The atlanto-axial joint is an example of this joint a. Ball and socket b. Saddle c. Wheel and axle d. Pivot e. hinge

d. Pivot

The following example illustrates a positive feedback mechanism: a. A decrease in blood sugar levels activates receptors that bring blood pressure below normal b. An increase in systemic blood pressure activates receptors in the carotid sinus which send signals to the brain, the final response being to bring blood pressures back to normal c. An increase in blood sugar levels trigger the release of insulin which in turn acts to bring sugar levels down to normal d. Platelets activated at the site of vascular injury release chemical messenger which attract more platelets to the same area e. Stretching the muscle spindle results in signals to the motorneuron such that all muscles of origin contract.

d. Platelets activated at the site of vascular injury release chemical messenger which attract more platelets to the same area

Conduction in the nerve axon has: a. Forward flow that is otherwise called an antidromic conduction b. Negative charges that is otherwise called orthodromic salutatory conduction c. Unmyelinated nerve axons called saltatory d. Positive charges that flow into an area of negativity called the current sink e. Backward flow that is otherwise called orthodromic conduction

d. Positive charges that flow into an area of negativity called the current sink

Junction formed by the terminal branches of the basilar artery and the two internal carotid arteries: a. Anterior communicating arteries b. None of these c. Circle of Willis d. Posterior communicating arteries

d. Posterior communicating arteries

In this lifting technique the individual establishes a wide, solid base of support with one foot positioned in front of the other. The body is slightly over the load with the knees bent in half squatting position. No bending at the waist takes place. In beginning the lift, the head moves first, followed by the straightening of the legs. a. Golfer's lift b. Deep squat lift c. Tripod lift d. Power lift e. Partial squat lift with support

d. Power lift

This is a form of indirect inhibition of a nerve cell: a. After hyperpolarization b. Renshaw inhibition c. IPSP due to interneuron d. Presynaptic inhibition e. Refractory period

d. Presynaptic inhibition

The following may cause biomechanical or physiological predisposition to microtrauma, EXCEPT: a. Foot malalignment that places abnormal stresses on particular tissues b. Insufficient flexibility c. Improper technique d. Proportional relationship between maximum output capacity and rapid expulsion of force e. Limb length discrepancies

d. Proportional relationship between maximum output capacity and rapid expulsion of force

The angle formed by the tendons of the quadriceps and ligamentum patella with the center a. Genu Valgum b. P Angle c. Genu Recurvatum d. Q Angle e. Genu Varum

d. Q Angle

Craniosacral therapists asses these aspects of craniosacral motion, EXCEPT: a. Symmetry, whether both sides of the skull demonstrate equal amplitude and rate b. Rate, the number of cycles of flexion per minute c. Quantity, whether the rate and corresponding amplitude is of a reasonable number d. Quantity, whether the motion is smooth or uneven e. Amplitude, the magnitude of excursion of flexion and extension

d. Quantity, whether the motion is smooth or uneven

The following levels of amputation are presently being performed, EXCEPT: a. Removal of arm from 7" above the elbow b. Removal of the foot at ankle joint c. Removal of lower arm above 7" below the elbow at the junction between the middle and lower third of the forearm d. Removal of leg 5" to 7" below the tibia e. Removal of patella with quadriceps brought over femur end

d. Removal of leg 5" to 7" below the tibia

Mitosis can only take place after this event takes place: a. Enzyme inhibition b. Negative feedback control c. None of these d. Replication of all DNA in the chromosomes

d. Replication of all DNA in the chromosomes

Inasmuch as the knee extensor mechanism is involved, the following statements apply, EXCEPT: a. The exercise is increased according to subject's tolerance b. Straight-leg exercises are progressed slowly c. By the time of discharge (about seven days), the subject should be doing 2-3 sets of 5 leg raises d. Resistance is seldom added to the exercise e. Repetitions should be aggressively increased until about 8 sets of 10 sets raises daily are performed over the next three weeks after discharge

d. Resistance is seldom added to the exercise

The following characteristics describe Type IIB muscle fibers, EXCEPT: a. Associated with mitochondria, myoglobin and sparse capillary density b. Fatigue rapidly c. High levels of glycolytic enzyme activity d. Resistant to fatigue e. Fast contraction times

d. Resistant to fatigue

These muscles retract the protracted scapula and turn the glenoid fossa downwards, thus forcibly lowering the raised arm, with other muscles, they keep the scapula applied to the chest wall. a. Serratus anterior b. Trapezius c. Pectoralis minor d. Rhomboids e. Levator scapulae

d. Rhomboids

A muscle's optimum angle of pull: a. Obtuse angle b. Acute angle c. 45 degrees d. Right Angle e. 180 degrees

d. Right Angle

The pulmonary artery leaves the heart via a. Left aortic b. It enters the heart c. Left ventricle d. Right ventricle e. Right auricle

d. Right ventricle

Stores and releases calcium ions during the contractile process a. Myofilament b. Tubules c. Myosin d. Sarcoplasmic reticulum e. Actin

d. Sarcoplasmic reticulum

The greater sciatic notch immediately below the sacro-iliac joint allows this nerve to pass: a. Gluteal nerve b. Femoral nerve c. Genitofemoral nerve d. Sciatic nerve e. Obturator nerve

d. Sciatic nerve

Hearing impairment resulting from damage to the inner ear or neural fibers of the eight cranial nerve: a. Peripheral b. Conductive c. Mixed d. Sensorineural e. Central

d. Sensorineural

The most powerful protractor of the scapula, its paralysis results in "winged" scapula: a. Levator Scapulae b. Pectoralis Minor c. Rhomboids d. Serratus Anterior e. Middle Trapezius

d. Serratus Anterior

The therapist is measuring shoulder external rotation. Proper positioning of the upper extremity in supine would best be describe as a. Shoulder abducted to 90 degrees, elbow flexed to 45 degree b. Shoulder in neutral, elbow flexed to 90 degrees c. Shoulder abducted to 90 degrees, elbow fully extended d. Shoulder abducted to 90 degrees, elbow flexed to 90 degree e. Shoulder in neutral, elbow fully extended

d. Shoulder abducted to 90 degrees, elbow flexed to 90 degree

Depolarization is initiated when this moves inside the cell. a. Potassium b. Magnesium c. Chloride d. Sodium e. Permanganate

d. Sodium

This muscle separates the neck into the posterior and anterior triangle and protects the great vascular and nerve trunks: a. Sternohyoid b. Scalenus anterior c. Sternothyroid d. Sternocleidomastoid e. Omohyoid

d. Sternocleidomastoid

The following statements characterize the fibula, EXCEPT: a. The upright triangle at the lower end of its medial aspect b. Act as a lateral "splint" for the ankle joint c. Has three joints with the tibia and one with the talus d. The fibula is the whole security of the ankle joint e. Act as pulley for tendons passing behind it at the ankle

d. The fibula is the whole security of the ankle joint

Which of the following statements is true of individuals without cardiopulmonary abnormalities? a. The increases in ventilation (VE) and CO are inversely proportional to the increase in metabolic rate, allowing venous blood gas and pH levels to remain close to baseline values during exercise b. The decrease in ventilation (VE) and CO are closely matched to the decrease in metabolic rate allowing venous blood gas and pH levels to remain close to baseline values during exercise c. The decrease in ventilation (VE) and CO are closely matched to the increase in metabolic rate, allowing venous blood gas and pH levels to remain close to baseline values during exercise d. The increases in ventilation (VE), and CO are closely matched to the increase in metabolic rate, allowing arterial blood gas and pH levels to remain close to baseline values during exercise e. The increase in ventilation (VE) and CO are closely matched to the decrease in respiratory rate, allowing arterial blood gas and pH levels to lower than baseline values during exercise

d. The increases in ventilation (VE), and CO are closely matched to the increase in metabolic rate, allowing arterial blood gas and pH levels to remain close to baseline values during exercise

The following statements apply to Translation, EXCEPT: a. A process by which a protein molecule is formed while the messenger RNA travels along the ribosomes b. A codon signals the end of a protein molecule c. The process is initiated when a molecule of messenger RNA comes in contact with a ribosome d. The manufacturing plant in which the protein molecules are formed

d. The manufacturing plant in which the protein molecules are formed

It is widely believed that these muscles are involved in "tennis elbow", a condition in which the extensor muscles, strained by an unusual amount of stress, cause pain at the lateral epicondyle of the elbow and along the course of the muscle. a. Extensor Communis b. Flexor Digitorum Superficialis c. Extensor Carpi Radialis Longus d. The mobile wad of three (Extensor Carpi Radialis Brevis in particular)

d. The mobile wad of three (Extensor Carpi Radialis Brevis in particular)

Following are example of anatomic pulleys, EXCEPT: a. In the leg, where tendons of the peroneal and long toe flexor descend distally on the posterior aspect of the legs and go behind and around the malleoli to form attachments in the foot and toes b. In the quadriceps and patellar tendons, where the direction of pull is changed as the knee flexed, and improves the force arm distance due to the interposed patella c. When the flexor digitorum superficialis contract, their tendons rise from the joint axes, and held down by loops that deflect the tendon with movement d. The patella raising the line of pull of the quadriceps muscle from the knee joint axis e. In the palmar side of the phalanges, when the flexor digitorum profundus contract their tendons rise from the joint axes, and held down by seven loops that deflect the tendon with movement

d. The patella raising the line of pull of the quadriceps muscle from the knee joint axis

Parkinson syndrome is a sign and symptom indicating this dysfunction a. Spinal cord b. Pyramidal c. Extrapyramidal d. basal ganglia e. cerebellar

d. basal ganglia

The following statements are true of the hips, EXCEPT: a. The anterior and lateral portions of the greater trochanter are covered by the tensor fascia latae and the gluteus medius muscle b. When the iliac crests are not level in relation to each other, it is usually because of pelvic obliquity c. When the greater trochanter is unequal, they may be due to a congenital dislocation of the hip or a hip fracture that has healed in a poor position. d. The pubic tubercles are just above the top of the greater trochanter. e. About three inches from the top of the iliac crest, the iliac tubercle marks the widest point on the crest

d. The pubic tubercles are just above the top of the greater trochanter.

Joint motion kinematic pattern is predictable during walking. The moments that influence these motions may be variable depending on the strategy used to achieve the k inematic result. The following describe these moments during the stance phase of gait, EXCEPT: a. The moment pattern at the knee is the least consistent lower extremity joint moment because moment at the ankle and hip can control movement at the knee when the foot is fixed on the ground b. Support moment has a shape similar to the vertical ground reaction force during stance c. Movement about the ankle has a consistently recognized moment pattern i.e., a small dorsiflexor moment followed by a large plantar-flexor moment d. The summation of the ankle plantar-flexor, knee extensor and the hip extensor moment remains inconsistent e. Moment pattern at the hip is less consistent than at the ankle, but usually shows an extensor moment followed by a flexor moment.

d. The summation of the ankle plantar-flexor, knee extensor and the hip extensor moment remains inconsistent

Shortly after the brachial plexus exit the vertebral bodies and pass between the scalenus anticus and medius muscles, the nerve roots of C5 and C6 join to form a. The middle trunk b. The posterior cord c. The lateral cord d. The upper trunk e. The lower trunk

d. The upper trunk

The following are characteristic of below-knee amputees, EXCEPT: a. Increase in gait speed is achieved by lengthening strides rather than by increasing the number of steps per minute b b. Loss of mobility at the ankle and foot, lack of distal muscular control, and absent proprioception all contribute to altered gait characteristics. c. These individuals have the potential for very high function because of their retention of normal hip and knee control d. There is substantial reduction in free walking speed, as a result of both diminished stride length and cadence e. Strength deficits in the remaining musculature often limit ambulation ability

d. There is substantial reduction in free walking speed, as a result of both diminished stride length and cadence

The following statements are true of the plantarflexors of the foot, EXCEPT: a. The tibialis posterior lies just posterior to the medial malleolus and is palpable as it comes around the medial malleolus and inserts into the navicular tubercle b. The flexor digitorum longus tendon lies immediately medial to the flexor hallucis longus tendon and is tested manually by stabilizing the calcaneus and asking the subject to bend or curl his toes c. The peroneous longus and brevis are the first two tendons posterior to the lateral malleolus d. To detect existing weakness of the gastrocnemius and soleus by muscle testing, the patient is asked to walk on his toes. If he is unable to perform the task, there is probably at least a minimal weakness in the calf muscle e. The flexor hallucis longus lies medial to Achilles tendon and to evaluate its function, simply observe the patient's gait.

d. To detect existing weakness of the gastrocnemius and soleus by muscle testing, the patient is asked to walk on his toes. If he is unable to perform the task, there is probably at least a minimal weakness in the calf muscle

Body parts moving in the same direction with equal velocity: a. Transfer b. Parallel c. Rotatory d. Translatory e. Angular

d. Translatory

This joint is primarily responsible for inversion/eversion and abduction/adduction of the foot. a. Talonavicular b. Talocrural c. Subtalar d. Transverse tarsal e. Calcaneocuboid

d. Transverse tarsal

These neurons carry motor impulses from the brain to the motor neurons in the spinal cord: a. Interneuron b. First-order neuron c. Lower motor neuron d. Upper motor neuron

d. Upper motor neuron

A therapist attempts to clear a patient's secretions after performing postural drainage techniques. What position would allow the patient to produce the most forceful cough? a. Supine b. Prone c. Half sitting d. Upright sitting e. Sidelying

d. Upright sitting

This great parasympathetic nerve supplies the thorax and abdomen. a. Abducens b. Hypoglossal c. Pelvic splanchnic d. Vagus e. Glossopharyngeal

d. Vagus

Ligaments function as follows, EXCEPT: a. Protect or hold in position some more important structure, usually a nerve b. When movements in a normal plane is gradually brought to a halt, the ligaments concerned at once become tensed as the movements proceeds c. Prohibits movement in undesired plane d. When abnormal movements are altogether prohibited the ligaments responsible at once are tensed and often violently e. Limit the range or extent of normal movement

d. When abnormal movements are altogether prohibited the ligaments responsible at once are tensed and often violently

The following statements are true of the brachialis, EXCEPT: a. It is an elbow flexor b. It is innervated by muscutaneous nerve c. Its proximal attachment is halfway up the shaft of the humerus d. When the forearm is supinated, the brachialis flexes the elbow with little participation from the biceps e. Attached distally to the coronoid process of the ulna

d. When the forearm is supinated, the brachialis flexes the elbow with little participation from the biceps

Muscle group affected in tennis elbow: a. Elbow extensors b. Elbow flexors c. Wrist pronator d. Wrist extensors e. Wrist flexors

d. Wrist extensors

The therapist examines a patient diagnosed with adhesive capsulitis. The examination reveals the patient has a significant capsular tightness in the anterior-inferior aspect. The most likely resultant range of motion limitation is a. Adduction and internal rotation b. Extension and external rotation c. Flexion and internal rotation d. abduction and external rotation e. flexion and external rotation

d. abduction and external rotation

The ligament/s which limit hip extension is/are: a. None of these b. Pubofemoral ligament c. Ischiofemoral ligament d. all of these e. iliofemoral ligament

d. all of these

During muscle contraction, they slide towards each other. a. A-band b. H-zone c. Myosin d. Z-discs e. Actin

e. Actin

A therapist observes a patient in the rehab waiting room that appears to be experiencing a heart attack. The most Significant sign of a heart attack is a. Dizziness b. Sweating c. Nausea d. chest pain e. shortness of breath

d. chest pain

Characterized by persistent posturing in one or more extremities, trunk, neck or face: a. Athetosis b. Spasticity c. Ballisimus d. dystonia e. chorea

d. dystonia

The anterior 1/3 is rostral to the central sulcus and above the lateral fissure on the lateral surface of the hemisphere: a. Occipital lobe b. Insular lobe c. Parietal lobe d. frontal lobe e. temporal lobe

d. frontal lobe

The primary deficit that CVA patient must learn to compensate to be able to return to driving is a. Dysarthria b. Homophobia c. Expressive aphasia d. hemianopsia e. agraphia

d. hemianopsia

To prevent both regurgitation and aspiration, the optimal position for patients with neuromuscular conditions but without increased intracranial pressure or gastric dysfunction is: a. Prone and Trendelenberg b. Ventral Trendelenberg c. Medial Trendelenberg d. lateral Trendelenberg e. Supine and Trendelenberg

d. lateral Trendelenberg

In the gate control theory of pain, stimulation of large fibers normally leads to inhibition of: a. Ventral horn cells b. Substanciagelatinosa interneurons c. Large diameter fibers d. small diameter fibers e. dorsal horn cells

d. small diameter fibers

The ability of the stomach to accommodate food and fluid without a significant increase in pressure is known as: a. Repetitive relaxation b. Perceptive relaxation c. retropulsion d. tone e. sieving function

d. tone

A therapist treats a patient status post CVA. What action would most likely facilitate elbow extension in a patient with hemiplegia? a. Turn the head to the unaffected side b. Flex the lower extremities c. Extend one of the lower extremities and the flex the upper opposing extremity d. turn the head to the affected side e. extend the lower extremities

d. turn the head to the affected side

This is an example of the nicotine acetylcholine receptor on the muscle end plate: a. Anionic current gated mechanism b. Active transport mechanism c. Voltage gated ionic channel d. Right Angle e. 180 degrees

e. 180 degrees

In a recent study conducted on phonophoresis, where hydrocortisone was prepared by blending hydrocortisone acetate with Aquasonic gel, which of the following statements is true? a. 5 minutes of hydrocortisone phonophoresis with 10% hydrocortisone using continuous ultrasound at 1.0 W/cm did not result in an increase in human serum cortisol level. b. 5 minutes of hydrocortisone phonophoresis with 10% hydrocortisone using continuous ultrasound at 1.0 W/cm resulted an increase in human serum cortisol levels c. 5 minutes of hydrocortisone phonophoresis with 10% hydrocortisone using continuous ultrasound at 5.0 W/cm did not result in an increase in human serum cortisol levels d. 10 minutes of hydrocortisone phonophoresis with 20% hydrocortisone using continuous ultrasound at 5.0 W/cm resulted in an increase in human serum cortisol levels e. 5 minutes of hydrocortisone phonophresis with 10% hydrocortisone using continuous ultrasound at 5.0 W/cm resulted in an increase in human serum cortisol level.

e. 5 minutes of hydrocortisone phonophresis with 10% hydrocortisone using continuous ultrasound at 5.0 W/cm resulted in an increase in human serum cortisol level.

Suggested dimension (in feet) for a treatment cubicle area is a. 10 x 10 b. 8 x 8 c. 6 x 8 d. 8 x 12 e. 8 x 10

e. 8 x 10

This chemical disconnects action from myosin and is hydrolyzed by the myosin molecule to produce the energy required for muscle contraction. a. Adenosine Diphosphatase b. Inorganic Phosphate c. Actomyosin Triphosphotase d. Adenosine Hydrolysis e. Adenosine Triphosphate

e. Adenosine Triphosphate

This vertically-directed device acts as the privet round which the rotation occurs and which brings this lateral rotation of the tibia (or medial rotation of the femur) to a halt. a. Lateral collateral ligament b. Posterior cruciate ligament c. Medial ligament d. Semilunar cartilages e. Anterior cruciate ligament

e. Anterior cruciate ligament

When using exogenous microcurrents, the type of tissue to be stimulated and the status of the tissue determine the polarity of the treating electrode. What type of stimulation would you use with an indolent ulcer? a. Either Cathodal or Anodal b. Cathodal and Anodal c. Anodal d. Not indicated e. Cathodal

e. Cathodal

Continuous discomfort may accompany many orthopedic conditions such as rheumatoid arthritis or osteoarthritis: a. Acute Pain b. Phantom Pain c. Superficial Pain d. Referred Pain e. Chronic Pain

e. Chronic Pain

Carpal Tunnel Syndrome can result in the following, EXCEPT: a. Compression can stem from anterior dislocation of the lunate bone or swelling secondary to Colles b. Constriction of the tunnel traps the tendons running through it, and restricts, and may even prevent flexion of the fingers c. Flexing the subject's wrist to its maximum degree and holding it in that position for at least a minute may reproduce tingling of the fingers (Phalen's Test) d. Pain is elicited or reproduced by tapping over the volar carpal ligament e. Compression of the median nerve can restrict motor function and sensation along its distribution in the lower arm.

e. Compression of the median nerve can restrict motor function and sensation along its distribution in the lower arm.

This structure is a modified skin that lines not only the inner surfaces of the lids but is also reflected on to the eyeball. a. Cones b. Canaliculus c. Tarsal gland d. Lacrima e. Conjunctiva

e. Conjunctiva

This ligament, in accompany with the Supraspinatus muscle, prevents the downward dislocation of the humeral head: a. Glenohumeral b. Coraco acromial c. Coracoclavicular d. Biceps e. Coracohumeral

e. Coracohumeral

Fascioscapulohumeral muscular dystrophy presents the following clinical picture, EXCEPT: a. Degree involvement ranges from facial weakness to severe generalized weakness b. Initially affects facial, scapular, upper arm, abdominal and foot dorsi flexor muscles c. Disease may progress to the proximal hip and distal arm and hand musculature d. Mode of inheritance is autosomal dominant e. Course is usually progressive but does not progress to wheelchair dependence

e. Course is usually progressive but does not progress to wheelchair dependence

The following statements describe ataxia, EXCEPT: a. Can result from damage to the dorsal and ventricular spinocerebellar pathways and pontine nuclei b. Can result from damage to the afferent portions of peripheral nerves and the dorsal column of the spinal cord. c. Can result from damage to several different motor or sensory regions of the central nervous system. d. Can result in high amplitude tremor that accompanies movement e. Damage in one cerebellar hemisphere is manifested contralateral to the lesion

e. Damage in one cerebellar hemisphere is manifested contralateral to the lesion

Galvanic current has gained attention in wound healing due to its predictable electric fields that form around the electrodes, attracting specific types of ions near the different poles. The following takes place under the anode, EXCEPT: a. Fibroblast migration b. Increased migration of epidermal cells, macrophages and leukocytes c. Protein synthesis d. Enhanced ion transport e. Decreased vascular congestion

e. Decreased vascular congestion

Which of the following examples therapeutically increase the pressure on a given area? a. Use of closed-cell foam shoe inserts for walking b. Prescribing a wearing schedule for a prosthesis c. Use of orthotics to increase the force arm length d. Frequent body weight shifts e. Decreasing the force arm length in orthotics

e. Decreasing the force arm length in orthotics

Splints that incorporate some elastic component such as springs of rubber bands that exerts a force on the joint but allow for some change of joint position. a. Dynamic-progressive b. Static-progressive c. Static-progressive d. Static e. Dynamic

e. Dynamic

Histologically, warm receptors in the skin are called a. Naked nerve endings b. Flower spray endings c. None of these d. Krause's end bulbs e. End organs of Ruffini

e. End organs of Ruffini

The state when the sum of forces acting on the body is balanced a. Terminal velocity b. Inertia c. Force Vector d. Gravity minimized e. Equilibrium

e. Equilibrium

Incorrect entries in your documentation may be edited by doing the following, EXCEPT: a. Corrected statement must be entered in the chronologic sequence b. On the margin, state why the correction was necessary c. Single line drawn through the inaccurate information, but still making it legible. d. Correction must be dated and initiated e. Erase or draw multiple lines on the inaccurate information.

e. Erase or draw multiple lines on the inaccurate information.

When terminating treatment, the caregiver a. Prepares and documents treatment plan and program. b. Documents client's program and linked to functional outcomes c. Establishes a functional diagnosis and outcome goals. d. Documents and re-evaluates the client's functional outcome and establishes and documents follow-up date. e. Evaluate patient's response to treatment and determines progress toward accomplishment of functional outcomes.

e. Evaluate patient's response to treatment and determines progress toward accomplishment of functional outcomes.

Glucose is primarily absorbed by this transport process a. Secondary active transport b. Primary active transport c. Complex Diffusion d. Simple Diffusion e. Facilitated Diffusion

e. Facilitated Diffusion

Entry of calcium into the cardiac muscle during the cardiac action potential is an example of: a. Secondary active transport mechanism b. Opening of ligand-gated channels c. None of these d. primary active transport mechanism e. opening of voltage-gated channels

e. opening of voltage-gated channels

Ergonomic design in office settings has become increasingly important particularly because of the increased use of computers. The following are acceptable recommendations, EXCEPT: a. Hard copy holder close to monitor to reduce eye motions and discomfort and allow proper neck posture. b. Keyboard placed at 15" from elbow height with a slight incline c. Padded wrist rest to reduce arm and shoulder discomfort. d. Top of monitor placed at eye level to allow proper head and neck position. e. Feet flat on the floor or footrest to provide stability

e. Feet flat on the floor or footrest to provide stability

A therapist examines a patient with cervical pain of unknown etiology. The therapist identifies shortening of the cervical spine extensor, upper trapezius and levator scapulae. The most probable postural deviation is a. Forward shoulders b. Kyphosis c. Scapular retraction d. Lordosis e. Forward head

e. Forward head

The following statements characterized postural drainage, EXCEPT: a. Refers to placing the body in a position that allows gravity to assist drainage of mucus from the lung periphery to the segmental bronchus and upper airway. b. The duration of postural drainage may range from 15 to 20 minutes, depending on the patient's tolerance to changes in position and the amount of sputum production c. Cooperative, spontaneously breathing patients who can cough effectively may not need postural drainage d. Enhances peripheral lung clearance, increases functional residual capacity and accelerated mucus clearances e. Fourteen positions are commonly used to drain 14 lung segments

e. Fourteen positions are commonly used to drain 14 lung segments

This structure is in the central part of the mediastinal part of the medial surface and is a large area where bronchi and pulmonary vessels plunge into the lung. a. Root b. Upper Sternal Angle c. Apex d. Mediastinum e. Hilus

e. Hilus

Which of the following statements describe the spleen? I. It is a contact with the under surface of the dome of the diaphragm, thus enjoying the protection of the lower liver II. It is about the size of a person's fissured lies in the upper left part of the abdominal cavity in contract with the diaphragm Ill. It regulates the number of the red blood cells in circulation IV. It maintains the amount of glucose present in the body at a constant level. V. It produces lymphocytes and is the principal residence of reticulo-endothelial cells of the body. a. I, III and IV b. III and V only c. II, IV and V d. I and III only e. II, III and V

e. II, III and V

The following statements are true in Trendelenburg test, EXCEPT: a. Congenital dislocation of the hip which brings the muscle's origin closer to its insertion may result in positive Trendelenburg sign b. When the subject is asked to stand on one leg and the stands erect, the muscle on the supported side should contract as soon as the leg leaves the ground and should elevate the pelvis on the unsupported side c. Designed to evaluate the strength of the gluteus medius muscle d. The subject may exhibit the characteristics Trendelenburg lurch if there is weakness of the gluteus medius e. If the pelvis on the supported side remains in position or actually descends, the muscle on the supported side is either weak or nonfunctioning

e. If the pelvis on the supported side remains in position or actually descends, the muscle on the supported side is either weak or nonfunctioning

The following statements describe the principle of all-or-non law, EXCEPT: a. Physiologic mechanism exist for fine gradation of the force of muscle contraction b. The law applies only to individual motor units c. All muscle fibers in a given motor unit contract or relax almost simultaneously d. Activated muscle fibers contract maximally if the nerve activates the motor fibers of a motor unit sufficiently e. In the same motor unit, it is possible for some muscle fiber to contract while others are relaxed

e. In the same motor unit, it is possible for some muscle fiber to contract while others are relaxed

Factors contributing to neuropathic plantar ulceration in patients with diabetes include the following, EXCEPT: a. Food deformities b. Limited mobility of the great toes metatarsophalangeal joint c. Limited mobility of the talocrural joint d. Limited mobility of the subtalar joint e. Increased plantar sensation

e. Increased plantar sensation

The gateway to pain in opened by a. Intense stimulation of large diameter fibers b. None of these c. Mild stimulation of small diameter fibers d. Mild stimulation of large diameter fibers e. Intense stimulation of small diameter fibers

e. Intense stimulation of small diameter fibers

Refers to the possibility that the conclusions drawn from experimental results may not accurately reflect what has gone on in the experiment itself. a. Parallel Reliability b. External Validity c. Low Reliability d. Incoherency e. Internal Validity

e. Internal Validity

The following conditions describe lesion of the cerebellum, EXCEPT: a. An oscillating tremor may appear on attempted movement b. Lesions of the vermis cause difficulty in maintaining upright posture, ataxia of gait and truncal ataxia c. Nystagmus is often present d. Acute cerebellar hemispheric lesion produces a tendency to fall to that side with weakness and hypotonia e. Intrinsics lesion produce sensory deficits

e. Intrinsics lesion produce sensory deficits

In thrombosis of the anterior spinal artery the following happens, EXCEPT: a. Produce bilateral atrophy at the level of the lesion b. Sudden onset of symptom with severe pain. c. Damage to spinothalamic tract results in loss of pain and temperature sense d. Produces flaccid paralysis at the level of the lesion. e. Involvement of bilateral corticospinal tract results in flaccid paraplegia

e. Involvement of bilateral corticospinal tract results in flaccid paraplegia

When force is applied at a site intermediate between the fulcrum and the weight to be moved, this lever a. Does not exist b. Is a first class lever c. Is a second class lever d. Can not be moved e. Is a third class lever

e. Is a third class lever

A polypropylene fixed or solid ankle-foot orthosis (AFO) is commonly prescribed to correct an equines gait pattern in children with spastic cerebral palsy. The following statements described this AFO, EXCEPT: a. Solid AFO covers the entire posterior calf and mediolateral borders and sole of the foot. b. Rationale of use is based on the inhibitive or tone-reducing cast. c. Reduces excessive ankle plantar flexion during stance. d. There are straps across the anterior upper tibia and front of the ankle e. It biomechanically controls the ankle by using a three-force system

e. It biomechanically controls the ankle by using a three-force system

The following statements describe the Ulnar Nerve, EXCEPT: a. The deep branch supplies the adductor Pollicis and all the Interossei. b. At about the midlength of the arm, it is found behind, and in contact with, the medial epicondyle of the humerus. c. The deep branch supplies the hypothenar muscles and the medial two lumbricals. d. The only muscles it supplies above the wrist are the Flexor Carpi Ulnaris and the medial half of the Flexor Digitorum Profundus. e. It divides into a superficial branch, a deep branch and an intermediate branch

e. It divides into a superficial branch, a deep branch and an intermediate branch

The following statements describe the Median Nerve, EXCEPT: a. Itis formed from the lateral and medial cords of the brachial plexus. b. It accompanies the axillary and brachial arteries. c. As it passes the cubital fossa, it gives off a deep branch, the anterior interosseous nerve d. It gives off branch only until it reaches the elbow region e. It enters with the long tendons and supplies the three thenar muscles and all lumbricals

e. It enters with the long tendons and supplies the three thenar muscles and all lumbricals

The following describe the modified Thomas test, EXCEPT: a. Hip ROM is considered normal if the patient has 80 degrees of knee flexion b. The pelvis is placed in approximately 10 degrees of posterior tilt by flexion of one hip and knee toward the chest while lying supine c. Hip ROM is considered normal if the patient has 10 degrees of femoral extension relative to the pelvis d. The other leg is positioned to extend off the edge of a treatment table far enough to allow the knee to bend freely e. It only tests the ROM and the length of the one-joint hip flexor muscles.

e. It only tests the ROM and the length of the one-joint hip flexor muscles.

Coordination disturbances are usually assessed through the following methods, EXCEPT: a. "finger-to-nose test" b. Reciprocal tracking tasks c. Tracking tasks d. Unilateral tapping tracking tasks e. Kinematic analysis

e. Kinematic analysis

A therapist is treating a patient with a dorsal scapular nerve injury. Which muscles would you expect to be most affected by this condition? a. Supraspinatus and infraspinatus b. Latissimus dorsi and teres motor c. Serratus anterior and pectoralis minor d. Serratus posterior and latissimus dorsi e. Levator scapulae and rhomboids

e. Levator scapulae and rhomboids

This hip-repair surgical procedure severes hip muscles and tendons around joint to decrease pressure and pain within the joint. The success rate is low and return to full weight status is a lengthy process: a. Displacement Osteotomy b. Arthrodesis c. Double-cup Arthroplasty d. Femoral Prosthesis e. Muscle Release

e. Muscle Release

Both peripheral and central nerve fibers have myelin sheath, but only peripheral nerve fibers additionally have: a. Synaptic transmitter b. Electrogenesis c. None of Ranvier d. Schwann cell e. Neurolemma

e. Neurolemma

Occupies an extensive area n the medial aspect of the hemisphere, this lobe of the lateral surface of the hemisphere is caudal to an arbitrary line drawn from the parieto-occipital fissures to the pre-occipital notch and a. Temporal b. Frontal lobe c. Insular lobe d. Parietal lobe e. Occipital lobe

e. Occipital lobe

Constricts the pupil and is involved in light and accommodation reflexes due to their connections with postganglionic neurons. a. Facial Nerve b. Trigeminal Nerve c. Optic Nerve d. Trochlear nerve e. Oculomotor nerve

e. Oculomotor nerve

When performing passive range of motion exercises to a quadriplegic in side lying, the most effective hand placement for hip extension is a. One hand at the heel, one hand supporting the knee b. One hand at the heal, one hand stabilizing the pelvis c. One hand at the heel, one hand supporting the mid thigh d. One hand and forearm at the heel and knee, one hand stabilizing the thing e. One hand and forearm supporting the leg at the knee, one stabilizing the pelvis

e. One hand and forearm supporting the leg at the knee, one stabilizing the pelvis

In the swollen ankle, pitting edema may be distinguished from synovial thickening in the following procedure. a. Excess synovial fluid and pitting edema are detected by pressing over the area b. They cannot be separately distinguished c. Pitting edema is assessed by balloting the fluid, excess synovial fluid is detected by pressing over the area d. Pitting edema and excess synovial fluid are both assessed by balloting the fluid e. Pitting edema is detected by pressing over the area; excess synovial fluid is assessed by balloting the fluid.

e. Pitting edema is detected by pressing over the area; excess synovial fluid is assessed by balloting the fluid.

A terminal lateral rotation of the tibia is said to "lock" the joint when the knee is fully extended. This key is the a. Gastrocnemius b. Biceps Femoris c. Piriformis d. Soleus e. Popliteus

e. Popliteus

The kinetic variable that reflects the rate of work performed at a given point. a. Torque b. Velocity c. Strength d. Energy e. Power

e. Power

The following are possible mechanisms of action for back belts, EXCEPT: a. Use of biomechanically safe lifting techniques as a result of proprioceptive input b. Decreased muscular fatigue and strain during lifting as a result of increased muscle support c. Decreased ROM as a result limiting spinal ROM d. Existence of a soothing effect as a result of increased local tissue temperature and a sense of safety e. Redistribution of spinal forces during lifting as a result of decreased intraabdominal pressure

e. Redistribution of spinal forces during lifting as a result of decreased intraabdominal pressure

Gait following stroke is grossly characterized by the following, EXCEPT: a. Lesser than normal knee flexion moment at mid-stance b. Sometimes loss of dorsiflexion of the ankle in the swing phase and initial contact c. Reduction or loss of the knee flexion phase in stance d. Increased stance on the unaffected side and decreased stance phase on the affected side e. Reduction of knee flexion in the swing phase

e. Reduction of knee flexion in the swing phase

The center of gravity at its lowest point is raised by: a. Knee motion during stance b. Adduction of femur c. Femoral abduction d. pelvic tilt e. pelvic rotation

e. pelvic rotation

At a comfortable room temperature (24-25 deg. C) the major mechanisms of heat exchange between the skin and environment is: a. Conduction, convection and evaporation b. Radiation, conduction and convection c. Radiation, conduction and evaporation d. evaporation, convection and conduction e. radiation, convection and evaporation

e. radiation, convection and evaporation

An attorney contacts you by phone and requests information on a patient he claims to represent. Questions asked include the extent of the patient's disability and their willingness to return to work. The most appropriate response would be to a. Tell the attorney not to bother you at work . b. Answer the question asked by the attorney c. Refer the attorney to supervisor d. Instruct the attorney to complete the necessary paperwork and a copy of the patient's therapy records will be sent to the appropriate party. e. Request the attorney to provide documented proof that he represents your patient and only then will you discuss the patient's situation

e. Request the attorney to provide documented proof that he represents your patient and only then will you discuss the patient's situation

Nerve cells at rest are positively charged on the outside and negatively charged on the inside a. Active potentials b. Action potential c. Hyperpolarization d. Depolarization e. Resting membrane potential

e. Resting membrane potential

Portion between two Z-discs a. H-zone b. I-band c. Y-band d. A-band e. Sarcomere

e. Sarcomere

The following interactions are true, EXCEPT: a. The muscles are only able to maintain weight and force if they are passively stretched by changing joint angle b. Physiological extensors that maintain body posture against gravity are activated more frequently than flexors c. Unloaded muscles lose weight and are less forceful even if they are exercised or stimulated for 20 minutes per day d. A muscle contracting at a long length would produce a greater load because of the passive length-tension properties of the muscle e. Shortening contractions in unloaded muscles will induce fatigue more rapidly than isotonic contractions because more oxygen is consumed.

e. Shortening contractions in unloaded muscles will induce fatigue more rapidly than isotonic contractions because more oxygen is consumed.

This nerve passes between the dense fascia of the forearm and the tendons of the brachioradialis and extensor carpi radialis longus muscles. The tendons can press on this nerve in a scissor-like fashion when the forearm is pronated, causing a proximal tethering on the distal segment of the nerve at the wrist, leading to entrapment causing pain patterns that can be mistaken for de Quervain's tenosynovitis: a. Posterior branch of the musculocutaneous nerve b. Ulnar nerve c. Dorsal antebrachial cutaneous nerve d. Median nerve e. Superficial radial nerve

e. Superficial radial nerve

The radial nerve dominant test is designed to place tensile stress on neural tissues of the upper limb along the course of the radial nerve. The following motions are applied to the upper limb with the patient in supine, EXCEPT: a. Medial rotation of the shoulder b. Shoulder girdle depression with approximately 10 degrees of shoulder abduction c. Elbow extension d. Ulnar deviation e. Supination of the forearm

e. Supination of the forearm

When a muscle contracts to eliminate some undesired movement that would otherwise be produced by the prime mover, it is said to be a/an a. Prime mover b. Fixators c. Antagonist d. Coordinator e. Synergist

e. Synergist

The following statements describe the medial compartment of the knee, EXCEPT: a. The pes anserine group crosses the posterior medial are of the tibia at the level of the tibial tubercle b. The semimembranosus muscle with its five branches is an important medial stabilizer that supports the posterior and posteromedial capsule and attach to the medial meniscus and tibia c. The posterior oblique ligament is important in controlling anteromedial rotator instability d. The posterior cruciate ligament tightens as the tibia internally rotates on the femur e. The anterior cruciate ligament, composed of posteromedial and anteromedial bundles is the main stabilizer of the knee

e. The anterior cruciate ligament, composed of posteromedial and anteromedial bundles is the main stabilizer of the knee

The following statements describe the Board of Examiners for Physical and Occupational Therapy, EXCEPT a. The Board is composed of a chairman and four members who shall be appointed by the President of the Philippines. b. The appointees are submitted to the President of the Republic of the Philippines through the Professional Regulation Commission. c. The chairman shall be a physiatrist and the four members shall be two professionally qualified physical therapists. d. The chairman and the members are appointed from the list of qualified professionals submitted by their respective bonafide national professional organization accredited by the Professional Regulation Commission. e. The chairman and the members of the Board are appointed from recommendation submitted by any qualified professionals

e. The chairman and the members of the Board are appointed from recommendation submitted by any qualified professionals

The osmotic gradient in the medullary interstitial is maintained because a. The vasa recta is permeable only to water and not to the solutes b. The oncotic concentration in the vasa recta increases from the cortex to the medulla c. The rate of fluid flow in the vasa recta is slower than the tubules d. The flow of fluid in the ascending extremities of the vasa recta is perpendicular but along the same direction e. The direction of fluid in the descending and ascending limb of the vasa recta is parallel and counter to each other

e. The direction of fluid in the descending and ascending limb of the vasa recta is parallel and counter to each other

The greater frequency of tears of the medial semilunar cartilage, a common athletic injury, is due to: a. The anterior cruciate ligament where the "screw-home" movement takes place and prohibits any father movement of either extension or rotation b. The posterior cruciate ligament which is attached to the tibia so far back as to allow some of its fibers to arise from the back of the bone below the upper surface c. The tightness of the medial coronary ligament of the knee that prevents back-and-fourth movements of the medial cartilage d. The aponeurotic insertion of the large extensor muscle of the knee that largely provides the fibrous capsule for the front and sides of the joint e. The medial meniscus which is fixed to the tibia front and back by its two horns as well as around its periphery

e. The medial meniscus which is fixed to the tibia front and back by its two horns as well as around its periphery

An example of this class of level is at the atlanto-occipital joint, where the head is balanced by the peel of neck extensor muscles a. Second b. Fourth c. Fifth d. First e. Third

e. Third

In performing this test, the patient with contracture will be unable to keep his leg, spine, and pelvis flat on the table when the opposite hip is flexed. a. Baker's test b. Segal's maneuver c. Patrick's test d. Gallic's flexion test e. Thomas test

e. Thomas test

Following are the many uses of electromechanical dynamometers, EXCEPT: a. To establish the relative efficacy of various treatment and training regimens b. To collect torque curves that might indicate whether pathology or characteristics specific to subject type were present c. To evaluate the effects of training or testing mode d. To quantify exercise so that exercise regimens may be administered e. To classify muscle performance as normal or abnormal by comparisons with the performance of ipsilateral muscles, with normal data, or with muscle performance in a control group.

e. To classify muscle performance as normal or abnormal by comparisons with the performance of ipsilateral muscles, with normal data, or with muscle performance in a control group.

The powers and duties of the Board of Physical and Occupational Therapy include the following, EXCEPT: a. To study the conditions affecting physical and occupational therapy education and prepare a written favorable recommendation to authorize schools to offer course in physical or occupational therapy b. To study, promote and improve the conditions of the practice of physical and occupational therapy c. To issue, suspend, revoke or re-issue any certificate of registration for the practice of physical and occupational therapy d. To give examinations to applicants for the practice of physical and occupational therapy in accordance with the provisions of the Philippine Physical and Occupational Therapy Law. e. To look into the conditions affecting the practice of PT and OT and delegate to accredited professional associations so they may adopt measures of good ethics and high professional standard in the practice.

e. To look into the conditions affecting the practice of PT and OT and delegate to accredited professional associations so they may adopt measures of good ethics and high professional standard in the practice.

The perpendicular distance from the pivot joint to the line of action of the weight is the: a. Mechanical advantage b. Force arm c. Lever arm d. Mechanical disadvantage e. Weight arm

e. Weight arm

The following statements describe clavicular motion, EXCEPT: a. After the shoulder has been abducted 90 degrees, clavicular transverse rotation occurs to complete scapular upward rotation and shoulder flexion b. The clavicle rotates at the sternoclavicular joint approximately 40 degree around its long axis. c. Clavicle motion include elevation-depression and protraction-retraction d. Tightening of the trapezoid and conoid ligaments cause the upward rotation of the clavicle e. When clavicular rotation is prevented, arm elevation is limited to 90 degrees.

e. When clavicular rotation is prevented, arm elevation is limited to 90 degrees.

Using proper body mechanics, which of the following statements is incorrect when moving an object? a. Keeping upper body straight, use the leg and hip muscles to stand upright, holding the object close to the body b. To move an object from table to floor, keep back straight, flex hips and knees and slowly lower the object to the floor c. If the object is too heavy, ask a coworker for help d. For an object placed on a high shelf, slide the object off the shelf and using a smooth coordinated movement, lower the object to a waist-level table e. When lifting the box at waist level, to change direction to the left, move your left foot so the toes are pointing left

e. When lifting the box at waist level, to change direction to the left, move your left foot so the toes are pointing left

The following ranges of motion may be attributed to the glenohumeral joint when the scapula is stabilized, EXCEPT: a. Flexing the elbow to 90 degrees isolates rotation of the glenohumeral joint from forearm supination and pronation. b. 40 to 60 degrees of hyperextension is available, limited by the superior and middle glenohumeral ligaments. c. Abduction can passively reach 120 degrees where it is then limited by the inferior glenohumeral ligament. d. Approximately 90 degrees of flexion take place. e. When the glenohumeral joint is externally rotated to 90 degrees, active abduction is limited to 90 degrees by active insufficiency of the brachialis muscles.

e. When the glenohumeral joint is externally rotated to 90 degrees, active abduction is limited to 90 degrees by active insufficiency of the brachialis muscles.

The following statements characterize thoracic outlet syndrome, EXCEPT: a. There is numbness in the fingers of the hand b. Involves both the nerves and adjacent blood vessels of the shoulder and the upper arm c. Arm may feel as if it is "going to sleep" d. There is compression of the nerves and blood vessels between the neck and shoulder e. Wrist pulse remain strong

e. Wrist pulse remain strong

The following can improve documentation, EXCEPT: a. Properly sign any entries entered on the chart with the caregiver's PRC license humber and expiration date. b. Writing must be legible and entries must be timely. c. Record significant information about the client's conditions, progress or response to treatment d. Use objective statements and provide continuity with status notes e. Your own abbreviation may be used so long as you inform the facility

e. Your own abbreviation may be used so long as you inform the facility

A 35 year-old male diagnosed with ankylosing spondylosis is referred in a home exercise program. A general treatment objective most beneficial for the patient will be to strengthen the a. Quadratus lumborum b. Quadriceps c. Internal and external obliques d. rectus abdominis e. back extensors

e. back extensors

Cell's permeability causes this rapid exchange of positive and negative ions: a. Action potential b. Repolarization c. Membrane potential d. resting potential e. depolarization

e. depolarization

These categories of nonorganic signs are used for patients with low back problems, EXCEPT: a. Hip abduction b. Shoulder extension c. Shoulder extension d. hip flexion e. elbow extension

e. elbow extension

A therapist observes a patient performing active hip abduction in supine. The patient appears to be moving through the full range of motion but the goniometric measurements taken previously indicated the patient was limited by 10 degrees. What hip compensatory measures might the patient use to seemingly increase hip abduction? a. Hyperextension and internal rotation b. Abduction with external rotation c. Hyperextension and external rotation d. flexion and internal rotation e. flexion and external rotation

e. flexion and external rotation

This characterizes diabetes mellitus. a. Decreased blood sugar b. Increase lipogenesis c. Androgen binding protein d. enhanced protein synthesis e. insulin deficiency

e. insulin deficiency

If there is posterior cruciate instability, the tibia will exhibit this movement as it is pushed back. a. Minor lateral movement b. Some anterior movement c. Extensive diagonal movement d. a lot of medial movement e. some posterior movement

e. some posterior movement

A therapist employed in a rehabilitation hospital utilizes the services of a therapy side. Which variable best determines the extent to which therapy sides are involved in patient care activities: a. The quantity of continuing education courses b. The number of years of experience c. Scope of formal training d. the aide must be young and strong e. the direction of the therapist

e. the direction of the therapist


Related study sets

AMSCO Chapter 20-25 quiz questions

View Set

Binary Numbers, Pure Binary, Binary Number Conversion, Negative Binary Number

View Set

Managerial Accounting Ch 6 Presentation

View Set

Chapter 14: Developing and Pricing Goods and Services

View Set

psych 201 final practice questions

View Set

Chapter 28: The child with a Gastrointestinal Condition

View Set

Chapter 3 - Interactions exposure

View Set

Google for Education - Calendar Practice Exam

View Set

Hunger Games: Sequence of Events

View Set

Ch. 06 Florida Statutes, Rules, & Regulations Common to All Lines.

View Set